You are on page 1of 69

A-PDF Merger DEMO : Purchase from www.A-PDF.

com to remove the watermark

Candidate Name: __________________________ CT Group: _______

Index number: __________

PIONEER JUNIOR COLLEGE


JC 2 Preliminary Examination 2009

ECONOMICS 9732/01
Higher 2

Paper 1 Friday 18 September 2009

2 hours 15 minutes

Additional materials: Writing Paper

READ THESE INSTRUCTIONS FIRST

Write your class, index number and name on all the work you hand in.
Write in dark blue or black pen on both sides of the paper.
You may use a soft pencil for any diagrams, graphs or rough working.
Do not use staples, paper clips, highlighters, glue or correction fluid.

Answer all questions.

At the end of the examination, fasten your work for Question 1 and 2 separately.

The number of marks is given in brackets [ ] at the end of each question or part question.

You are advised to spend several minutes reading through the data before you begin writing your
answers.
You are reminded of the need for good English and clear presentation in your answers.

This document consists of 9 printed pages

[Turn over
2
Answer all questions.

Question 1

The Aviation Industry

Extract 1: Deregulation of America’s domestic airline market

Since the deregulation of America’s airline market in 1978 which led to greater competition in the
airline market, fares have fallen by half in real terms. Fares have been brought down chiefly by the
arrival of low-cost and no-frills airlines, either directly or by putting pressure on established carriers.
But, there have been other factors too. Companies have cut back on travel and restricted the use
of first class and business class.

In the five years to 2005 America’s airlines lost a cumulative $35 billion. The older ―legacy‖ carriers
such as United, American Airlines and Continental, have been slashing costs and staff to compete
with the newer discount operators, such as Southwest and JetBlue. In 2005 the number of airline
jobs in America fell to its lowest level since 1995. Although passenger numbers are growing, on
the whole Americans are spending less on air travel. In the two decades to 2000 the average
annual revenue from passenger flights accounted for 0.95% of America’s GDP. This began to
decline even before September 11th 2001 and has now dropped to 0.73%. This represents $32
billion a year in lost revenue. This could force more mergers among airlines. However, according
to Mr Baggaley of Standard & Poor’s, a credit-rating agency, there are risks of trying to put
together disparate businesses and working cultures. But there is one bit of cheer, at least for
airlines: the supply of seats in the American domestic market is not likely to outpace demand. As
long as capacity remains tight, prices can be nudged higher.

Figure 1: United States Air Fares and Inflation

Source: The Economist Print Edition, 14 June 2007


3
Extract 2: Aircraft emissions

Airlines are accused of having a free ride in terms of air pollution because they pay no tax on the
fuel they used for international flight. Even though today’s aircraft are about 70% more efficient
than those of 40 years ago, concerns over emissions have grown. Aircraft emissions could be
limited by restricting flying. That could be achieved by imposing higher taxes because a large part
of the market (mostly leisure) is price-sensitive.

Many in the industry, however, are convinced that by using more sophisticated technology and
more efficient operating procedures they can make air travel a lot cleaner still. Not unreasonably,
they also point to the huge economic benefits that airports and airlines bring. Some studies
suggest that aviation may account for as much as 8% of global GDP when you allow for tourism,
which is possibly the world’s biggest industry, and air cargo, which is now responsible for some
40% of inter-regional exports by value.

As there is in the immediate future little alternative to jet aircraft fuelled by some form of kerosene,
aviation is one of the most costly area in which to cut carbon dioxide emission, says Brian Pearce,
IATA (International Air Transport Association)’s chief economist. Emission trading is a possibility. It
wants an open and global system so that airlines can trade permits to emit carbon with industries
that can achieve reductions in more cost-effective ways, such as factories or power stations
anywhere in the world.

Carriers have already been hit by terrorism, war, recession, the respiratory disease SARS and
soaring oil prices. There were hopes the industry could make a small profit in 2007, but having to
pay for environmental costs could change that. Yet global warming is not something that airlines,
or any other airlines, can shake off for ever.

Figure 2: Causes of US greenhouse-gas emission, %

Source: The Economist Print Edition, 14 June 2007


4
Extract 3: Prying open ASEAN’s skies

A new agreement reached by the Association of Southeast Asian Nations (ASEAN) will allow
regional air carriers to make unlimited flights among the grouping’s 10 capital cities as early as
December 2008. The deal promises to boost regional tourism, trade, and aviation related
investments, some industry analysts say.

Some Asean countries have started to reap benefits from liberalizing their air market. For example,
Malaysia has seen a boom in air-traffic growth due to greater domestic competition led by the
region’s largest and most successful low-cost carrier: Air Asia. According to the Pacific Asia Travel
Association (PATA), Malaysia’s domestic and international passenger growth has soared since Air
Asia’s market entry in 2004, with traffic to Thailand alone growing on average 37% annually. When
Malaysia and Thailand agreed to allow Air Asia to commence international flights, each country
experienced annual economic benefits of about US$114 million. Singapore currently has the most
liberalized aviation sector in ASEAN. In recent years, air-travel-related services have contributed
as much as 9.2% to the island’s gross domestic product, according to a National University of
Singapore report.

Overall, intra-ASEAN air services remain highly restricted by protectionistic policies aimed at
insulating domestic air carriers from foreign competition. For example, the Singapore-Kuala
Lumpur route has for years been restricted to protect Malaysia Airlines Services (MAS)’s previous
monopoly over the lucrative domestic market. The new agreement has the potential to change all
that and usher in a new era of cheap intra-regional air travel. However, greater regional
competition for ASEAN’s opening aviation market will no doubt create new winners and losers.
Domestic carriers in ASEAN’s poorer countries, including Myanmar, Cambodia, Laos and Vietnam,
are expected to fare poorly when faced with greater regional competition.

Source: Edward Russell, Asia Times Online, 2007

Table 1: IATA Global Traffic Forecast Table 2: International Traffic by Region


International Flight 2008-2009
Passenger Freight Growth Passenger Growth
Growth Africa -9.2%
2007 7.6% 5.3% Asia/Pacific -12.0%
2008 2.4% -2.6% Europe -7.6%
2009 F -8.0% -17.0% Latin America -3.2%
2010 F 0.4% 3.5% Middle East 7.1%
2011 F 4.8% 7.7% North America -8.9%

Source: IATA
5
Questions

(a) Using Figure 1, explain why airfares in America have fallen in real terms since deregulation
in 1978. [2]

(b) Extract 1 indicates that although passenger numbers are growing, on the whole Americans
are spending less on air travel. Explain why this has happened. [3]

(c) Explain the likely value for the price elasticity of supply of air travel in America. [2]

(d) Comment on the actions taken by the established airlines to compete with the budget airlines.
[5]

(e) Discuss whether taxation on pollution by airlines is justified. [8]

(f) Assess the benefits of liberalisation of the airline market for Singapore and the other ASEAN
members. [10]

[Total: 30]

[Turn over
6
BLANK PAGE
7
Question 2

Economic Performances of Germany and France

Extract 4: The German economy

In the past few years the main motor of growth has shifted. When the economy was at its weakest,
in 2002 and 2003, exports kept it running. They continue to do well—in manufacturing exports.
Germany is in the right businesses at the right time: China, India, Russia and other countries in
central and eastern Europe are growing fast and wanting the goods in which Germany specialises
– machine tools. Consumer confidence has rebounded. Consumers are being buoyed by the
labour market.

Germany has held down wages for six years. Germany has regained international competitiveness
- hence the success of its exporters. Those in work are reaping the benefits of years of belt-
tightening and unemployment is falling rapidly. As companies have come under more pressure,
workers' pay has been squeezed: as higher returns on capital have been demanded, the relative
price of labour has had to fall. Managers can threaten to move work to central Europe or even
farther away has strengthened capital's hand. As a consequence, while company profits have risen
steeply, workers have done much less well. Real wages have barely risen at all.

Adapted from Frankfurt and Leipzig Jul 12th 2007 The Economist print edition

Extract 5: The French economy

France's poor labour market performance is well known and has been a problem for decades. But
until recently the economy was propped up by low interest rates and robust consumer spending.
Wages have continued to rise, helping support consumer spending but gradually leading to a loss
of competitiveness that is now becoming painfully apparent. Their problem has become a complete
lack of competitiveness in overseas markets due to labour market rigidities and strong wage
growth. What France probable needs are: agreement from the unions on a more flexible work
contract, on minimum service standards during strikes in public services

Throwing public money at the problem, which is the traditional urge of French politicians, will be
difficult since the country is running a budget deficit of 2.5% of GDP - well above the eurozone
average of 1.6% - and has debt levels of 64% of GDP. With the state already accounting for half of
GDP, the public-spending route now looks exhausted. Reform of the labour market and public
finances are not enough, France needs growth. The only comfort is that many of Europe's biggest
companies are French and they have highly productive workforces. The Paris region, in particular,
is becoming a magnet for global IT and services companies and is sucking in record amounts of
inward investment.

Adapted from Ashle Seager, The Guardian, 7th May 2007


8
Table 3: Germany’s Macroeconomic Indicators

Germany 2003 2004 2005 2006 2007


Inflation rate (%) 1.5 1.6 1.2 2.0 2.3
Unemployment rate (%) 9.3 9.8 10.6 9.8 8.4
Real GDP growth (%) -0.2 1.2 0.8 3 2.5
Current A/C balance (% of GDP) 2 4.7 5.2 6.1 7.5
Current A/C balance (Billion Euro) 47.3 77.9 95.8 116.1 181.9
Government Budget (% of GDP) -4 -3.8 -3.3 -1.5 0.1
FDI (US$ m) 32398 -10195 41943 55199 50932
Expenditure on health (% of GDP) 10.8 10.6 10.7 10.6 NA
Life expectancy at birth (years) 78.6 79.2 79.4 79.8 NA

Table 4: France’s Macroeconomic Indicators

France 2003 2004 2005 2006 2007


Inflation rate (%) 2.1 2.0 1.8 1.6 1.5
Unemployment rate (%) 9 9.3 9.3 9.2 8.3
Real GDP growth (%) 1.1 2.5 1.9 2.2 2.2
Current A/C balance (% of GDP) 0.8 0.6 -0.6 -0.7 -1.2
Current A/C balance (Billion Euro) 10.3 -4.4 -30.9 -32.6 -28.5
Government Budget (% of GDP) -4.1 -3.6 -3 -2.4 -2.7
FDI (US$ m) 42538 32585 84887 78198 157974
Expenditure on health (% of GDP) 10.9 11.0 11.1 11.0 NA
Life expectancy at birth (years) 79.3 80.3 80.2 80.9 NA

Source: www.oecd.org
9
Questions

(a) (i) Describe the trend in the government budget balance as a percentage of GDP in
Germany between 2003 and 2007. [2]

(ii) Compare this trend with the change in the France’s government’s budget balance as a
percentage of GDP in the same period. [2]

(b) Extract 4 indicates that wages in Germany are kept low. Explain a possible reason for
Germany’s unemployment rates shown in Table 3. [2]

(c) With reference to the data, account for the difference in the current account balance between
France and Germany. [6]

(d) Discuss whether the data are sufficient to assess changes in the standard of living in these
economies over the period. [8]

(e) With reference to the data, evaluate the measures that France can adopt to achieve its
macroeconomic aims. [10]

[Total: 30]

End of Paper
Suggested Answer

(a) Using figure 1, explain why airfares in America have fallen in real terms
since deregulation in 1978. [2]

As the nominal airfares have risen at a slower rate than the CPI, real airfares
have fallen. (2)
Optional: Real price measures the price after excluding the effects of inflation
(which is measured by the consumer price index). (1)

2 marks for explanation of why real price fell


1 mark if student merely define real price but gave the wrong explanation.

(b) Extract 1 indicates that although passenger numbers are growing, on


the whole Americans are spending less on air travel. Explain why this
has happened. [3]

Although air fares (prices) have fallen due to increased competition from the
budget carriers, quantity demanded may have increased less than
proportionately due to price inelastic demand. (2) Thus, expenditure (price X
quantity) on air travel by Americans has fallen. (1)

1 mark for identifying that Q rises due to P fall


1 mark for identifying a less than proportionate rise in Q
1 mark for linking P and Q to change in total expenditure

If students merely says that SS rises, price falls and hence TE falls – max 1
mark

(c) Explain the likely value for the price elasticity of supply of air travel in
America. [2]

Price elasticity of supply is a measure of how much the supply of a product


changes when there is a change in the price of the product. From the last line
in Extract 1, it is evident that there are capacity constraints in the provision of
air travel and this implies that the supply of air travel could be price inelastic,
having a value between 0 to 1.

If students gave an accurate definition of PES but inaccurate explanation and


value of PES – max 1 mark
If students are able to give some plausible explanation for PES (elastic or
inelastic) that is not from the passage – max 1 mark
To score max 2 marks – must be able to use data but do not need to give
definition and do not need to give the absolute value of PES

1
(d) Comment on the actions taken by the established airlines to compete
with the budget airlines [5]

The entry of budget airlines have increased competition and driven prices
down. Therefore, in order to maintain its competitiveness and market share,
the established airlines have responded by merging with other established
airlines or by slashing costs and staff.

When firms merge to create one new business, they are able to gain a greater
market share and market power as they gain internal economies scale by
sharing resources and information. The reaping of substantial economies of
scale allows established airlines to enjoy cost savings which increases their
cost competitiveness so that they could compete with the budget airlines.

However, if firms get too large as a result of a merger, diseconomies of scale


may set in. Problems such as managerial diseconomies of scale and a
possibility of culture clash may lead to a fall in productivity, conflicts and
underperformance.

Furthermore, airlines have streamlined their organization by slashing costs


and retrenching staff. This reduces their costs of production and allows them
to be more cost competitive and gives them an avenue to restructure and
realign themselves with the current economic climate.

However, the costs of adopting cost cutting measures must be taken into
account. Firstly, attempt to cut wages by airlines could fail because of
resistance from the strong trade unions. Secondly, there could be morale
costs as those not affected by the slashing of staff may experience some form
of grief as they fear for their own jobs constantly and this may ultimately affect
their productivity. Secondly, firms may incur some form of “brand equity” costs
as they embark on cost cutting measures. When companies now all state an
explicit goal of being an “employer of choice”, the war for talent, especially in
a knowledge based economy is extremely stiff. Therefore, a company’s brand
as an employer must be considered before deciding on any cost cutting
measures. Last but not least, firms may incur higher rehiring costs when the
economy picks up and they are not able to train and hire factors of production
such as labour fast enough to meet the increase in demand.

At least 2 well explained and analysed actions


Identifying and explaining only 1 action – up to 2 marks
Identifying and explaining 2 actions – up to 3 marks
Analysis and evaluation – 2 additional marks

2
(e) Discuss whether the taxation on pollution by airlines is justified. [8]

The pollution by airlines is a form of negative externality that imposes external


costs that are damaging to third parties. In the airlines industry, the pollution is
not taken into account by the airline companies and therefore it results in
market failure, which refers to the inability of markets to allocate resources in
an optimal manner and maximize community surplus.

The presence of pollution created by air travel could be internalized by


imposing a tax equivalent to MEC on airline companies. This would converge
the MPC towards MSC, achieving social efficiency. This is essential because
when firms only consider their private costs, there would be a divergence
created between MPC and MSC due to the presence of MEC, which refers to
the environmental costs in this context. Thus, there is an overconsumption of
air travel where too much air travel is consumed at too low a price resulting in
the cost imposed on society outweighing the benefits, thus leading to a
welfare loss. With a tax equivalent imposed on airline companies, it would
make the airline companies pay for the pollution and reduce the provision of
air travel (since market is price sensitive according to Extract 2, Line 5) as a
result of the higher cost.

Taxation is a beneficial measure because it provides incentives for the airlines


to reduce pollution so as to reduce their burden of taxation. In addition, the tax
revenue collected by the government can be used to reduce the pollution
caused by the airlines and to fund research into cleaner fuel and more
efficient aircraft, thus helping to reduce pollution in the long run.

However, there are some limitations to taxation. Firstly, the MEC cannot be
easily quantified and measured. In addition, it is difficult to determine the
amount of pollution each airline company is producing and to what extent
each airline company is responsible for the pollution.

Secondly, according to Extract 2 Paragraph 3, air travel has little substitutes


and is one of the most costly areas to reduce pollution. Hence, despite the
increase in the cost of air travel due to the tax and due to a price inelastic
demand for air travel as a result of a lack of substitutes, air travel may fall less
than proportionately and the intention to reduce air travel may not materialize.

Thirdly, aircraft only accounted for 3.2% of total pollution in US. Thus, by
reducing aircraft emissions through taxation, total pollution will still remain
high.

In addition, if air travel does fall due to the increased cost of travelling, it may
create other problems such as a fall in income and productivity when

3
businesses and trade are being restricted and constrained. With many airlines
already badly hit by other external shocks such as terrorism, rising oil prices
and recession, further increase in cost due to tax on pollution may cause
many airlines to close down resulting in falling national output and
employment, depending on the contribution of air travel to total output. From
Extract 3, it is mentioned that the airline industry plays a more influential role
in terms of contribution to a country’s economic growth in recent years. The
increase in the price of air travel due to the tax may dampen consumption and
this would mean a contraction of the entire airline industry as a whole,
ultimately leading to falling national incomes and subsequently, causing the
balance of payments to worsen and unemployment rate to rise.

Hence, according to Extract 2, the implementation of tradable permits could


be a better solution instead. This could be a cost effective measure for
reducing air pollution as industries which are able to reduce pollution at a
lower cost will do so, thus minimising the cost of reducing pollution. However,
tradable permits may cause airlines to shelve their measures to reduce
pollution as they could always buy the permit in the market whenever they
pollute excessively.

Despite its limitations, taxation may still be an effective measure in the long
run as it forces airlines to find alternative fuels to reduce pollution. This is
particularly important with the growing concern of aircraft emissions in future.
By finding more efficient and cleaner fuel, airlines will also be less affected by
rising oil prices in future.

Level Descriptors Marks

2 Able to evaluate the policy of taxation with good 7 – 8


use of data

Able to evaluate the policy of taxation with some 5 – 6


or no use of data

1 Able to explain how taxation works to reduce 3 – 4


pollution by airlines

Weak explanation with inaccuracies 1–2

A discussion with no application to airlines – maximum 4 marks

4
(f) Assess the benefits of the liberalization of the airline market for
Singapore and the ASEAN members [10]

The liberalization of the airline market means that entry of airline companies
are no longer being controlled and prices are no longer subjected to approval.
This would lead to increased competition between airlines, depressing prices
on air travel and therefore having a substantial influence on efficiency, trade,
national income levels and employment of the affected nations.

Benefits for Singapore

Microeconomic Benefits

Singapore International Airlines (SIA) being one of the most established


airlines in South East Asia will stand to gain the most. With the liberalization
of the airline market, Singapore is able to gain access and venture into newer
and larger markets. This would mean that the revenue of SIA is likely to
increase and with a higher amount of profits, SIA is able to invest in planes
with bigger carrying capacity (such as the Airbus) and improve on its service
standards through retraining and more efficient management strategies. A
larger and more competitive SIA will also enable her to benefit from the
expected increase in global traffic as shown in Table 1.

The ability of airline companies to reap benefits of external economies of


scale would allow firms to be more cost competitive as well as more
productive efficient. Coupled with the greater competition, this will weaken the
monopoly power of the large international airlines and hence price of air travel
will fall (as evident in the US market) to its marginal cost leading to allocative
efficiency and greater welfare gains by consumers.

However, the liberalization of the airline market would also mean that
competition is likely to increase within the industry causing total revenue of
international airlines to fall as was the case in the US airline market. SIA,
facing greater competition together with rising costs stemming from rapid
growth to our economy, will need to constantly restructure and improve on its
quality of service in order to keep up with the rise of budget airlines. SIA also
has to constantly upgrade itself in the area of product innovation so as to
differentiate itself from the rest of the international carriers. Only when SIA is
able to achieve a successful method of differentiating itself in terms of
branding and quality of service provided can we maintain our status as the top
notch airline carrier in South East Asia.

5
Macroeconomic benefits

The benefit of gaining access to the regional market would mean that SIA
may need to employ more factors of production to meet this increase in its
scale of operation. With the employment of more factors of production such as
labour, incomes are expected to rise and coupled with the higher export of
airline services, Singapore’s national income is expected to rise by a
multiplied amount through the multiplier effect. From Extract 3, it is mentioned
that the liberalization of the Asean airline market has led to higher traffic
growth and this is a clear reflection of an expansion of the entire airline market
as a result of liberalization.

As a result of a liberalized market, the opening up of more flight routes and


increased investment by all air carriers would imply that consumption of air
travel has become cheaper and this is likely to increase passenger numbers
across the entire region. Hence, tourism revenue and the increase in
consumption of flight services are likely to improve our balance of payments
account, promoting further growth and economic development. However, the
liberalization of the airline market would also mean that air carriers are now
more prone to external negative shocks and will make our national carrier
more vulnerable and our profits more dependent on the performance of our
regional neighbours. The volatility in the airline industry is reflected in the
sharp negative fall in the international traffic in Asia Pacific region as
compared to the other regions in Table 2.

Benefits for ASEAN members

Liberalization of the air market would lead to increased passenger and trade
flows within ASEAN itself and this may pave the way for more investments
and development to take place. ASEAN therefore, may experience greater
growth and with a more integrated region in the areas of trade and co-
operation, ASEAN is also well positioned to compete with the rest of the major
superpowers such as USA and Germany on the global arena. With a stronger
bargaining power, ASEAN should be able to negotiate for more FTAs (or open
sky policies) with the other regions, improving our terms of trade with other
nations and eventually result in further growth of the region as a whole. This
would lead to the creation of more employment and investment opportunities
within ASEAN and overall income level throughout the region should see a
positive impact too.

However, as the region gets more connected and integrated as an economic


and political entity, problems of coordination and equity may come into play as
the benefits of liberalization is often not evenly distributed throughout the
entire region. Weaker Asean countries such as Laos and Cambodia may lose

6
out as currently their domestic airlines may not be strong enough to compete
with the stronger foreign airlines. Thus, some form of protection and support
may have to given to the weaker Asean carriers so that they do not lose out
from the greater competition.

Level Descriptors Marks

2 Clear explanation of the microeconomic and 4 – 6


macroeconomic benefits for Singapore and Asean

1 Weak and limited explanation of the benefits for 1 – 3


Singapore and Asean

E1 Unexplained evaluation of the extent of benefit 1–2

E2 Well supported evaluation with use of data 3–4

7
1
JC2 H1 Preliminary Exam 09 – Macro Case Study

Question 2 Economic Performances of Germany and France

Extract 4: The German economy

In the past few years the main motor of growth has shifted. When the economy was at its weakest,
in 2002 and 2003, exports kept it running. They continue to do well—in manufacturing exports.
Germany is in the right businesses at the right time: China, India, Russia and other countries in
central and eastern Europe are growing fast and wanting the goods in which Germany specialises
– machine tools. Consumer confidence has rebounded. Consumers are being buoyed by the
labour market.

Germany has held down wages for six years. Germany has regained international competitiveness
- hence the success of its exporters. Those in work are reaping the benefits of years of belt-
tightening and unemployment is falling rapidly. As companies have come under more pressure,
workers' pay has been squeezed: as higher returns on capital have been demanded, the relative
price of labour has had to fall. Managers can threaten to move work to central Europe or even
farther away has strengthened capital's hand. As a consequence, while company profits have risen
steeply, workers have done much less well. Real wages have barely risen at all.

Adapted from Frankfurt and Leipzig Jul 12th 2007 The Economist print edition

Extract 5: The French economy

France's poor labour market performance is well known and has been a problem for decades. But
until recently the economy was propped up by low interest rates and robust consumer spending.
Wages have continued to rise, helping support consumer spending but gradually leading to a loss
of competitiveness that is now becoming painfully apparent. Their problem has become a complete
lack of competitiveness in overseas markets due to labour market rigidities and strong wage
growth. What France probable needs are: agreement from the unions on a more flexible work
contract, on minimum service standards during strikes in public services

Throwing public money at the problem, which is the traditional urge of French politicians, will be
difficult since the country is running a budget deficit of 2.5% of GDP - well above the eurozone
average of 1.6% - and has debt levels of 64% of GDP. With the state already accounting for half of
GDP, the public-spending route now looks exhausted. Reform of the labour market and public
finances are not enough, France needs growth. The only comfort is that many of Europe's biggest
companies are French and they have highly productive workforces. The Paris region, in particular,
is becoming a magnet for global IT and services companies and is sucking in record amounts of
inward investment.

Adapted from Ashle Seager, The Guardian, 7th May 2007


2
Table 3: Germany’s Macroeconomic Indicators

Germany 2003 2004 2005 2006 2007


Inflation rate (%) 1.5 1.6 1.2 2.0 2.3
Unemployment rate (%) 9.3 9.8 10.6 9.8 8.4
Real GDP growth (%) -0.2 1.2 0.8 3 2.5
Current A/C balance (% of GDP) 2 4.7 5.2 6.1 7.5
Current A/C balance (Billion Euro) 47.3 77.9 95.8 116.1 181.9
Government Budget (% of GDP) -4 -3.8 -3.3 -1.5 0.1
FDI (US$ m) 32398 -10195 41943 55199 50932
Expenditure on health (% of GDP) 10.8 10.6 10.7 10.6 NA
Life expectancy at birth (years) 78.6 79.2 79.4 79.8 NA

Table 4: France’s Macroeconomic Indicators

France 2003 2004 2005 2006 2007


Inflation rate (%) 2.1 2.0 1.8 1.6 1.5
Unemployment rate (%) 9 9.3 9.3 9.2 8.3
Real GDP growth (%) 1.1 2.5 1.9 2.2 2.2
Current A/C balance (% of GDP) 0.8 0.6 -0.6 -0.7 -1.2
Current A/C balance (Billion Euro) 10.3 -4.4 -30.9 -32.6 -28.5
Government Budget (% of GDP) -4.1 -3.6 -3 -2.4 -2.7
FDI (US$ m) 42538 32585 84887 78198 157974
Expenditure on health (% of GDP) 10.9 11.0 11.1 11.0 NA
Life expectancy at birth (years) 79.3 80.3 80.2 80.9 NA

Source: www.oecd.org
3
Questions

(a) (i) Describe the trend in the government budget balance as a percentage of GDP in
Germany between 2003 and 2007. [2]

(ii) Compare this trend with the change in the France‘s government‘s budget balance as a
percentage of GDP in the same period. [2]

(b) Extract 4 indicates that wages in Germany are kept low. Explain a possible reason for
Germany‘s unemployment rates shown in Table 3. [2]

(c) With reference to the data, account for the difference in the current account balance between
France and Germany. [6]

(d) Discuss whether the data are sufficient to assess changes in the standard of living in these
economies over the period. [8]

(e) With reference to the data, evaluate the measures that France can adopt to achieve its
macroeconomic aims. [10]

[Total: 30]
4
Suggested Answers

(a) (i) Describe the trend in the government budget balance as a percentage of GDP in
Germany between 2003 and 2007. [2]

From Table 3, the government budget balance as a percentage of GDP in Germany has
improved [1] from deficit to a surplus [1].

Alternative answers that are accepted:


 only mentioning of ‗deficit to surplus‘ [2m] or
 mentioned that the budget is in deficit throughout the period, except for 2007 [2m]

Those candidates who described budget as ‗negative‘/‘positive‘ will not be given credit.

(ii) Compare this trend with the change in the France‘s government‘s budget balance as a
percentage of GDP in the same period. [2]

(Tables 3 & 4) Compare with France, both countries‘ budget balances have improved [1].
However, France’s budget balance remains in deficit while Germany’s budget
balance has improved to a surplus position, at the end of the period [1].

Alternative answer accepted as refinement:


 Germany‘s budget improves faster than France‘s.

(b) Extract 4 indicates that wages in Germany are kept low. Explain a possible reason for
Germany‘s unemployment rates shown in Table 3. [2]

Additional explanation of question to student:


While wages are low (Extract 4), the unemployment rates (Table 3) seem fairly high.
However, according to market forces (for labour), low wage would induce higher qty
demanded for workers (by firms), ceteris paribus. Hence the data may not be explained by
simple DD/SS theory.

A possible way of explaining for the data could be that the high unN (despite low wages) may
arise more from ss-side reasons, such as structural unN or frictional unN (ie ss-side unN or
voluntary unN). Thus, even when wages are low, there are still workers who lack the skills or
are in search for better job, and remain as unemployed.

Other possible reasons for the high unN, even when wages are low:
 According to the data, the use of capital or capital-intensive production results in
the low dd for labour (capital as a substitute for labour), then unN will still be high
(ie firms prefer to use capital than labour for production) although wage is low.
 There could be an influx of workers from other European Union (EU) member
countries into Germany (as under this economic integration, FOPs, especially
labour is free to move among the member countries). Then Germany‘s natural
rate of unN would be higher.
 If unN benefits are high in Germany, this can encourage more to remain
unemployed and claim for the unN benefits. Then unN rate would still be high.

1m – identify a possible reason


1m – explain the reason proposed.
5

(c) With reference to the data, account for the difference in the current account balance between
France and Germany. [6]

Current balance (shown from both % of GDP and Billion Euro) for France deteriorated from
surplus to a deficit, while Germany’s current account surplus improved over the years.
For Germany‘s case, trade seemed to become more and more an important sector in
influencing the country‘s GDP.

According to Extract 4, Germany‘s improving current account surplus may be explained by


her export competitiveness (rise in export lead to improvement in current balance). This is
achieved by maintaining low wages and hence production costs. Thus Germany‘s exports
can be priced competitively in the international markets. Moreover, her comparative
advantage in producing capital goods (―machine tools‖ in Extract 4) is helping her to export
significantly to developing countries (such as China, India and Russia mentioned in the
extract). With high demand and competitively-priced exports, Germany is able to enjoy
improving current account surplus.

Based on Extract 5, France‘s worsening current account can be explained by the robust
consumer spending which may encourage consumers to import more. In addition, France is
losing her export competitiveness as high wages drives up production cost. With more
imports and deteriorating exports (or export competitiveness), France current account
position can worsen, ceteris paribus.

Lastly, the data only suggest trade or visible accounts as possible reasons for the countries‘
current account positions. No information about the invisible trade accounts is available to
provide more evidences for the current account positions of Germany and France in Tables 3
and 4.

1m – identify the difference


2m – explain for Germany‘s current account position
2m – explain for France‘s current account position
1m – qualify for the lack of data in explaining for the difference between the 2 countries‘
current account positions

(d) Discuss whether the data are sufficient to assess changes in the standard of living in these
economies over the period. [8]

Standard of living (SOL) reflects the material and non-material well-being of the residents in a
country. Some of the macroeconomic indicators presented in Tables 3 and 4 can be used to
indicate the material and non-material well-being of Germany and France respectively.
Together with relevant information mentioned in extracts, the SOL of Germany and France
may be studied.

According to the data, Germany is enjoying economic growth from 2004 to 2007 (based on
real GDP growth rates shown in Table 3). This would mean that Germans are experiencing
higher income which can be used to improve their material and non-material well-being.
However, Extract 4 mentions that real wages do not rise significantly when firms try to stay
competitive by keeping wages low. Hence the extent of SOL improvement may be limited.
Nonetheless, the slowing down of the unemployment rate in 2007, the significant expenditure
on healthcare (more than 10% of GDP) and the high life expectancy are evidences to support
that Germany‘s SOL has improved over the period (2004-2007). However to stop at such
6
conclusion relying only on these information may seem too naive because more data to
accurately reflect SOL is needed. For instance, information about income distribution,
working hours, stress-related issues which hint the quality of life (non-material well-being) is
not available. If majority of the income is earned by the minority, then we cannot conclude
that the average German‘s SOL has improved. Additionally, if the economic growth is
attained by Germans‘ long and stressful work efforts (ie production replaces current
consumption), then Germans may actually experience lower quality of life. Moreover, if the
high unemployment rates in Table 3 signals structural unemployment, then significant
number of Germans may suffer from lower material and non-material well-being, instead of
an improving SOL as indicated superficially by the data given. The SOL in Germany may
further be worsened if the economic growth is achieved at the expense of a deteriorating
environment (eg. pollution or destruction to the eco-system).

As for France, based on similar nature of indicators (as that used to examine Germany‘s SOL
above) shown in Table 4, positive real economic growth rates may be the first step towards
studying France‘s SOL. With higher income, together with encouraged consumer spending
(Extract 5), material well-being of France seems to improve SOL. This is especially so if
consumers in France indulge in better-quality imports or enjoy a wider range of consumer
goods that enhance their material and non-material well-being. In addition, the substantial
expenditure on healthcare and high life expectancy also hint that SOL has improved over the
years (similar to that of Germany). Furthermore, with the help of strong unions, French
workers may not be subjected to a competitive work environment (eg less stressful work or
shorter working hours), and their fringe benefits may be secured to improve their quality of
life. Again, more data on income distribution and those reflecting non-material well-being
need to be used before arriving at a more accurate conclusion about France‘s SOL. For
instance, if the bulk of the income is earned by those in the IT and service sectors, then it
may not be accurate to conclude that an average person in France is enjoying higher SOL.
The production of certain ‗bads‘ such as drug abuse and environmental damage may render
a lower SOL as indicated originally by the data.

Given the limited data presented in Tables 3 and 4, it seems that Germany‘s SOL is on par
with France‘s SOL, and that they (SOL) are improving steadily and slowly over the years.
However, Germany‘s SOL may be more sustainable than France‘s as the underlying
economic problems (rigid labour market, loss of export competitiveness, unhealthy budget
deficit and national debt) may deny France‘s current SOL (to worsen it in the future).
Although Germany has more favourable economic situations (export competitiveness and
better budget position compared to France), the improvement in SOL may be limited if the
keen foreign competition reduces the non-material well-being. These conjectures, based on
the limited data available, may change if other data mentioned earlier is considered.

Level Descriptor Mark range


2 a well-balanced discussion that considers both thesis and anti-thesis 5 - 8m
(limitations of data in concluding the SOL) with good knowledge of
SOL shown.
1 limited discussion, reflecting a lop-sided view and/or with errors that 1 - 4m
show misconception (limited understanding) of SOL

(e) With reference to the data, evaluate the measures that France can adopt to achieve its
macroeconomic aims. [10]

Based on Table 4 to consider the 4 macroeconomic aims of France, inflation rate is lower
than Germany, so it seems that it (inflation) does not pose a problem to France. However if
7
the robust consumer spending (encouraged by low interest rates and relatively high wages)
persists causing demand-pull inflation, GPL may rise more in the future and would attract the
government‘s attention.

Unemployment rates in France are comparable to Germany‘s, but Extract 5 hints labour
market rigidities and wage growth. Hence the unfavourable unemployment rates and the
long-term implication to export competitiveness are causes of concern and the French
government needs to address.

Growth rates, although positive and seem somewhat sustained (in Table 4) in the past years,
may not prevail in the future with the loss of export competitiveness. With Extract 5 echoing
the need for France to promote growth, this is one macroeconomic objective the government
cannot neglect.

It is apparent that the unfavourable current account position (Table 4) and the lack of
competitiveness (Extract 5) warrant government‘s intervention to remedy this
macroeconomic problem.

Then, the next question is which macroeconomic policies can help France to address these
problems above. According to Extract 5 and Table 4, persistent budget deficit is not
sustainable (the cumulated nation debts amounting to 64% of GDP) and may not be adopted
once again to manage the problems above. Even if the government resorts to rely on fiscal
policy again, the emphasis is more on influencing AS than AD (so that dd-pull inflation will not
develop, and the rise in employment and growth rates are more sustainable in the long term).
Moreover using fiscal policy to manage AS can help in gaining export competitiveness to
correct for the unfavourable current account position. Possible ways to carry out such policy
would be to reduce corporate tax (not income tax so that dd-pull inflation can be avoided and
also not increasing govt spending to add on to the current debt burden), introducing tax
incentives to those firms which strive to reduce production cost and/or enhance
productivity/efficiency especially in the export sector and those which can develop new
comparative advantage through R&D. Although such measures slow down the govt‘s attempt
to reduce the national debt burden, it may help to address the current macroeconomic
problems and control the debt from rising further. The tax incentive may also attract more
investment to boost AD and AS (in the long run), which help in promoting growth,
employment and enhance export capability. [With more firms engaging in production, more
tax revenue can be collected (although with lower corporate tax rate, the tax collected from
each firm is lower, the increase no. of firms would compensate for the lower tax rate imposed,
to end up with higher tax revenue at the end of the day). This could also help to address the
high debt burden and the persistent budget deficit.]

On the other hand, as mentioned in Extract 5, it may be difficult to garner support to


implement wage or labour market reforms to make the labour force more flexible and nimble,
so as to produce goods and services in high demand (in the export market). Thus specific
supply-side policy to improve labour efficiency can be a challenge. Nevertheless, growth
strategy is proposed in Extract 5, hence a calibrated supply-side policy to target on
capitalising French‘s comparative advantage(s) may help France in achieving the
macroeconomic aims. According to Extract 5, France‘s comparative advantages seem to be
IT and services and they are attracting substantial investment. So, a variation to the
education/training initiative can be considered to send a clear signal of this govt emphasis to
the households and firms. Such supply-side policy may help to control inflation; promote
long-term economic growth, employment and favourable BOP position. However, Govt‘s
investment in infrastructure may be limited since the budget deficit is a concern at present
(based on Extract 5).
8

While the last type of (economic) policy option is the monetary policy. Exchange rate policy
may not be considered as France is a member country of European Union which circulates
the common currency, Euro. Moreover the interest rate is currently low, it would not be
effective to adopt expansionary monetary policy by lowering the interest rate further
(especially if she is experiencing liquidity trap). Changing money supply is difficult too as the
supply of currency (Euro) is not merely determined by the French govt. alone. [H2 students
can bring in a short discussion about the forgoing the option of controlling money supply
when fixed ER system is adopted as hinted by the common currency, Euro.]

In sum, the French govt has limited policy options to address the macroeconomic problems
on hand (unN, growth and unfavourable BOP). After eliminating monetary policy, the govt is
left with mainly ss-side policy (which may not gain the support of workers) and fiscal policy
(again, limited due to the huge budget deficit and national debt problem). Nevertheless, these
feasible policies are plagued with limitations/difficulties. The silver lining seems to rise from
French comparative advantages in IT and services, thus effective policies to target on
making Paris to be Europe‘s ‗Silicon Valley‘ and a service hub may help in dealing with the
current economic problems.

Level Descriptor Mark range


2 a discussion showing good content knowledge which is applied 4 – 6m
appropriately to France‘s macroeconomic situation and the policies
that can be adopted (ie in consideration of data).
1 a vague response to the question, showing error in understanding of 1 – 3m
content knowledge and/or not answering clearly to the question on
France‘s macroeconomic problems and policy options (ie not relating
to data or context). Alternatively, the discussion could be restricted to
merely a theoretical one which ignores the data/context given.
E2 an attempt with explained judgement 3 – 4m
E1 a response with unexplained judgement 1 – 2m
Candidate Name: __________________________ CT Group: ______

Index Number: ______

PIONEER JUNIOR COLLEGE


JC 2 PRELIMINARY EXAMINATION 2009

ECONOMICS 9732/02

Higher 2

Paper 2 Wednesday 16 Sept 2009

2 hours 15 minutes
Additional materials: Answer paper

READ THESE INSTRUCTIONS FIRST

Do not open this booklet until you are told to do so.

Write your name, CT group and index number in the spaces provided on the cover page and
all the answer papers provided.

Begin each section on a fresh sheet of paper. At the end of the examination, you will be
required to hand in your answers to the different sections separately.

Attach the cover page to answer for Section A.

Answer three questions in total, of which one must be from Section A, one from Section B
and one from either Section A or Section B.

The number of marks is given in brackets [ ] at the end of each question or part question.
You are reminded of the need for good English and clear presentation in your answers.
The number of marks is given in brackets [ ] at the end of each question or part question.

This paper consists of 2 printed pages inclusive of this page


2

Answer three questions in total.

Section A
One or two of your three chosen questions must be from this section.

1 “Motorists face more misery at petrol pumps as oil prices hit record high. The
government has proposed to use price controls and subsidies to curb the
problem.”

(a) Explain why the price of oil would increase. [10]

(b) Discuss the effects of government’s intervention in the petrol market on


the economy. [15]

2 (a) Using appropriate diagrams, compare price determination in a perfect market


with that of imperfect market. [10]

(b) Discuss whether the conduct of firms that operate in an imperfect market is
necessarily harmful to society. [15]

3 The objectives of government microeconomic policy are to achieve efficiency in


resource allocation and to promote equity in income and wealth distribution.

Assess the various policy measures that the Singapore government currently adopts
to achieve the above microeconomic objectives. [25]

Section B
One or two of your three chosen questions must be from this section.

4 (a) Explain the effects of an increase in investment by firms on output and


prices in an economy. [10]

(b) Discuss how the Singapore government influences private investment to


ensure sustained economic growth. [15]

5 A record level of retail spending in May has fanned fears of higher interest rates.

(a) Would you support the implication of this statement that inflation is caused by
consumer spending? [10]

(b) Discuss how effective interest rates alone might be in controlling inflation if the
rate of inflation were to become unacceptably high. [15]

6 Explain the effects of globalization on the Singapore economy and assess


whether the Singapore government currently adopts appropriate policies to
minimize the costs of globalization. [25]
Prelim Micro essays

1 “Motorists face more misery at petrol pumps as oil prices hit record high. The
government has proposed to use price controls and subsidies to curb the
problem.”
Adapted from Daily Mail 26/10/2007

(a) Explain why the price of oil would increase. [10]


(b) Discuss the effects of government’s intervention in the petrol market on
the economy. [15]

(1) Analysis of Question (to be done in 1-2 mins)


“Motorists face more misery at petrol pumps as oil prices hit record high. The
government has proposed to use price controls and subsidies to curb the problem.”
Adapted from Daily Mail 26/10/2007

(a) Explain why the price of oil would increase. [10]

Command word: Key word:


Explain - requires you Increase -
to select and apply Key word:
Price – to look at Note to talk about a
economic concepts and rise in the price of
principles (in this case demand and supply
factors that would oil
dd/ss analysis) to
analyse the rise in price cause the price to
of oil increase

Question requirement
This question requires you to clearly explain how the causes (dd and ss factors)
lead to the effect (increase in oil price), (Causal Link)

(2) Schematic Plan (to be done in 2-3 mins)

Increase in oil price

Increase in demand Fall in supply

- global expansion - depletion of oil resources


- rising national income in - political tension in the Middle
emerging countries East has limited oil production
- expectations of future - OPEC cut back SS to increase
increase in prices price

Price of oil increase

1
(3) Write out the full essay (to be done by following closely to the outline in 15 mins)

Thinking Process Content / Analysis / Evaluation


Introduction: The price of a commodity like oil is determined by demand
Define key concepts and supply conditions. Therefore, changes in demand and
supply conditions will cause changes in the price of oil. A rise
in the price of oil can either be caused by an increase in
demand for oil or a decrease in supply for oil or both.
Referring to figure 1 below, the initial price is at P1. A rise in
demand for oil would cause the demand curve to shift right
from D to D1 while a fall in supply would cause the supply
curve to shift left from S to S1, resulting in a new equilibrium
price of P2 which is higher.

Give appropriate Price of S1


diagram as tool of oil
analysis (in US$)
S
P2
Figure 1
P1

D1
D
Quantity

Body: Factor 1 – The global expansion has seen strong economic growth in
Global expansion many countries. The global economic growth has led to an
(DD) increase in demand for fuel to power production in most
countries. As a country‟s output and production increase, the
demand for a raw material like oil would increase as well since
oil is needed to run the machinery in the plants and factories.
Rising national income and strong economic growth would in
turn induce optimism in investors who would increase their
investment, thus increasing production and the demand for oil.
These would thus lead to an increase in the demand for oil,
causing the demand curve to shift right.

Body: Factor 2 – The world has also witnessed the phenomenal growth of
Rising national emerging countries like China and India in the recent years.
income and The rising income in such emerging countries, especially with
industrialisation (DD) the expansion of the middle class, would result in a rising
demand for private transport like cars, which is a luxury good
and is likely to be income elastic. The rise in income would thus
result in a more than proportionate increase in the demand for
cars. This would lead to rising demand for petrol, which in turn
drives up demand for oil. Rapid industrialisation in these
countries has also caused production to increase, thus
increasing the demand for oil, causing the demand curve to
shift right.

Body: Factor 3 – Expectations about the future changes in the price of oil
Speculation (DD) would also affect its demand now. If speculators expect the
prices of oil to increase in the future, they would buy more oil
2
Thinking Process Content / Analysis / Evaluation
now and increase the demand for oil. This is so that when the
price of oil does increase in the future, these speculators
would be able to make a profit by selling their stocks. The
increase in demand would thus cause the demand curve to
shift right.
Body: Factor 1 – Besides demand factors, changes in supply would also affect
Depletion of oil the price of oil. The continuous extraction of oil might have
resources (SS) resulted in a depletion of oil sources as oil is a non-renewable
resource. Bad weather conditions like hurricanes and typhoons
might also have destroyed oil refinery facilities. These would
cause the supply of oil to fall and the supply curve to shift left.
Body: Factor 2 – Must of the oil production in the world takes place in the
Political tension (SS) Middle East. The violence and the political tension taking
place in the Middle East would have disrupted or limited oil
production in these countries. Such security concerns may
increase the COP or hamper the production of oil, causing a
fall in supply and the supply curve to shift left.
Body: Factor 3 – OPEC is the international cartel which controls the supply of
OPEC (SS) oil and thus supplies oil to the rest of the world. OPEC might
be reluctant to increase production levels in order to maintain
high prices for oil. Alternatively, OPEC might be cutting back
or hoarding the supply of oil to push up the price. The
increase in price of oil would increase their total revenue as
the demand for a raw material like oil is inelastic and thus an
increase in price would also result in a less than
proportionate fall in the quantity demanded.
Conclusion In conclusion, the rise in the price of oil is a combination of both
demand and supply factors. However, the recent increase in
the price of oil is largely demand driven due to the huge
increase in demand of oil from countries like China and India.
The supply factors serve to exacerbate the increase. The
demand for oil is fairly inelastic as it is a raw material needed in
production processes and its supply is inelastic as well since it
takes time and a lot of effort to extract oil. With inelastic PED
and PES, the increase in price of oil is even more significant.

Level Descriptor Marks


L3 Clear explanation of both demand and supply factors causing 7-10
the increase in oil prices and application of elasticity concepts
to explain price changes. (2D and 1S factors or 2S and 1D
factors)
L2 Explanation of demand and supply factors increasing prices 5-6
but explanation is not contextual. (At least 1 demand and 1
supply factor)
L1 For an answer that shows some knowledge that prices are 1-4
determined by demand and supply intersection. Mere listing of
the demand and supply factors.

3
(b) Discuss the effects of government’s intervention in the petrol market on
the economy. [15]

(1) Analysis of Question

“Motorists face more misery at petrol pumps as oil prices hit record high. The
government has proposed to use price controls and subsidies to curb the
problem.”
Adapted from Daily Mail 26/10/2007

(b) Discuss the effects of government’s intervention in the petrol market on


the economy. [15]

Command word Key word Key word


Discuss - requires Effects – to Government’s
you to examine the intervention – need
Key word
- use economic to explain how price
Economy – need impact on
concepts to stakeholders controls and
to examine how
explain the effects subsidies afffect the
govt‟s policies
on the relevant price of oil and its
would affect the
stakeholders impact on the
stakeholders
- evaluate the economy
impact on
economy of govt‟s
actions

(2) Schematic Plan

Effects

Price ceiling Subsidies

Consumers Price of petrol falls, Price of petrol falls and


increases CS qty increases, increases
CS
Fuel-producing Revenue is hurt as price Benefit as COP falls
firms and qty falls
Government Burden on govt‟s budget
Economy Distorts the price Might result in more
mechanism externalities due to
congestion and pollution

4
(3) Write out full essay

Thinking Process Content / Analysis / Evaluation


Introduction: The increase in petrol prices due to the increase in oil price
Rationale for would affect a large proportion of the population as petrol is
government‟s essential to both households and firms. Households would
intervention require petrol if they own a vehicle while firms require petrol
for transportation purposes. As such, the rise in petrol prices
might increase the cost of living and lower the SOL of the
citizens. Hence, the government proposed to use price
controls and subsidies to try to reduce the price of petrol.
Body: Explain how a To bring down the price of petrol, the government would have
price ceiling works to impose a price ceiling. Price ceiling is the maximum price set
by the government below the market equilibrium price which is
deemed too high. Referring to Figure 2, in order to protect the
consumers from paying a higher price at the equilibrium price
PE, the government sets a maximum price PMAX below the
equilibrium price PE. At price PMAX, consumers pay a lower
price. However, a shortage is created since quantity demanded
exceeds the quantity supplied at PMAX. Though consumers now
pay a lower price for petrol, the quantity of petrol transacted
has fallen to Qs.
Price

S
Figure 2
Pb

PE

Maximum
PMA Price
X
Shortage D
Quantity
QS QE QD
To resolve the shortage, the government may need to impose
rationing – allocate the good through non-price rationing
schemes like coupons or first-come-first-serve basis. However,
this may lead to the emergence of a black market to fulfil the
needs of people who do not get the good. At output 0Qs,
consumers are willing to pay a maximum price of 0Pb. Thus the
maximum black market price is 0Pb.

Effects on consumers The lower price of petrol will benefit consumers of petrol in
terms of affordability and increase their consumer surplus.
However, consumers who have to resort to the black market to
obtain their petrol would suffer, especially if they belong to the
lower income group, as the price is much higher. Moreover,
while price controls are often used by government with good
intentions of improving equity, they prevent the price
mechanism from clearing the market, resulting in shortages
which cause inefficient allocation of resources.

5
Thinking Process Content / Analysis / Evaluation
Effects on firms The firms selling petrol will be hurt under the price ceiling
scheme as the price and quantity transacted has fallen. Their
total revenue would thus fall. Such firms might also be less
motivated to invest in oil extracting and processing processes
as their profit margins are squeezed. The lack of R&D and
innovations by such firms might hinder the long run economic
growth of the economy.

Body: Explain how a Besides imposing a price ceiling, the government can also give
subsidy works subsidies to reduce the price of petrol. A subsidy will cause the
cost of production to fall and the supply of petrol to rise. Thus, a
subsidy causes the supply curve of petrol to shift to the right
from S to S1. As seen from figure 3, this will result in a fall in
price from Po to P1 and an increase in quantity from Q2 to Q1.
The fall in price of petrol would increase consumer surplus. For
a given change in supply, the price would fall by a greater
extent for an inelastic demand compared to an elastic demand.
Thus, a subsidy is more beneficial to consumers in this case as
they will enjoy a greater price fall since the demand for petrol is
inelastic due to it being a necessity with few substitutes.

Price S
0 S1
P0

P1

Di

Q2 Q1 Qty
Figure 3

Impact on consumers When subsidies are adopted, consumers benefit from both
lower prices and higher consumption level. However, the
subsidy will make the consumers more dependent on
government support and delay their switch to alternative
sources of fuel. Such consumers will suffer if the government
decides to remove the support schemes in the long term.
Impact on economy Subsidies require financing from the government and hence
might be a strain on the government‟s budget. Such
subsidies might also cause a burden on taxpayers. An
opportunity cost is also incurred when the government
spends on petrol subsidies as less funds are available for
other areas like healthcare and education which can help to
promote long term growth of the country.

By depressing the price of petrol, petrol becomes more


affordable. This may result in more externalities due to traffic
congestion and pollution as there could be more usage by the
consumers, thus increasing the market failure. Moreover, as
consumers over-consume and producers over-produce when
the price of petrol is artificially low, it would result in a wastage
6
Thinking Process Content / Analysis / Evaluation
and misallocation of economic resources. The over
consumption might add to the global demand for fuel, putting
pressure on the price to increase.

Macro impact If petrol takes up a significant proportion of consumers‟


expenditure, the fall in the price of petrol might cause a fall in
inflation. The fall in inflation in the economy might in turn
stimulate production and investment as cost of production has
fallen due to a fall in the price of fuel. The increase in
investment would increase AD and via the multiplier process,
would increase the national income by multiple times.
Employment would also increase as an increase in output
would increase the demand for labour. The extent of the
increase in national income would depend on the size of the
multiplier.

Conclusion In conclusion, price control and subsidies implemented by the


government would benefit some while causing others to suffer.
Taken together as an economy, the impact of the government
intervention is generally bad as it will lead to welfare loss and
also distortion to the price mechanism, leading to misallocation
of resources. There could also be an exacerbation of the
problems of externalities. Thus the government should only
implement such measures in the short term to help its people
cushion the impact of rising petrol prices and should focus on
spending more resources and money on R&D to look for
alternative sources of fuel and greener technology.

Level Descriptor Marks


L3 Clear discussion on the effects on the various stakeholders, 9-11
considers both positive and negative effects, with reference to
the context
L2 Explanation of measures which can be adopted to reduce 6-8
prices but lacks development or examine only positive or
negative effect with only some relation to the context.
L1 For an answer that shows some knowledge of how price 1-5
controls and subsidies work. Mere listing of effects.
E2 For a conclusion with sound justification and evaluation on the 3-4
impact of government intervention. (Eg: Comparison of the 2,
trade off between efficiency and equity when using price
ceiling, price ceiling is good for the government as budget is
unaffected.)
E1 For an unexplained conclusion on the effects of government 1-2
intervention.

7
2 (a) Using appropriate diagrams, compare price determination in a perfect
market with that of imperfect market. [10]

(b) Discuss whether the conduct of firms that operate in an imperfect market
is necessarily harmful to society. [15]

(1) Analysis of Question (to be done in 1-2 mins)

(a) Using appropriate diagrams, compare price determination in a perfect market


with that of imperfect market. [10]

Command word: Key word:


Compare - requires Key word: Perfect
you to distinguish Key word: Key word: Imperfect market– to
pricing decisions of Diagrams Price market– to look at
firms under perfect and – to draw determination look at pricing pricing
imperfect markets, and – to look at decisions of decisions of
pointing out the explain pricing MC/Monopoly firms in PC
similarities and diagrams decisions of and oligopoly market
differences firms firms structure

Question requirement
This question requires you to clearly explain and compare how prices are
determined under perfect and imperfect markets.

(2) Schematic Plan (to be done in 2-3 mins)

Price determination

Perfect competition (PC) Imperfect market (Focus:


monopoly)

price is determined by both buyers and sellers


(Profit maximising condition: MC=MR)
- price taker - price setter due to market
power
- P = MR - P > MR
- price determined by market - output determined by MC=MR
demand and supply condition, price is then read from
demand curve

8
(3) Write out the full essay (to be done by following closely to the outline in 15 mins)

Thinking Process Content / Analysis / Evaluation


Introduction: The behaviour of firms is affected by the market structure in
Define key concepts which the firm operates in and the level of competition of the
market structure. The type of market structure affects how the
firm makes its price and output decisions. At one extreme is
perfect competition (PC), which is a perfect market, where
there are many small firms selling homogeneous products and
firms are free to enter the industry due to a lack of barriers to
entry. Thus, the firm has no power to control price and is a
price taker. At the other extreme is monopoly, where there is
no competition as there is only one firm in the industry selling a
unique product. Thus, the firm has significant market power to
control price. In between are the monopolistic competition
(MC) and oligopoly structures. In the monopolistic competitive
market, although there are many firms, each firm sells
differentiated products thus giving each firm some market
power in controlling the price of its product. The oligopoly has
greater market power as the few large firms are able to limit
competition by restricting new firms from entering the market.
Monopoly, MC and oligopoly all fall under imperfect market. To
study the pricing behaviour of firms, it is assumed that firms
aim to maximize profits.

Body: Similarity Regardless of the market structure that the firm operates in,
between firms in the price is determined by both buyers and sellers in the
perfect and imperfect markets. The demand curve will illustrate the willingness and
competition ability to pay of the consumers while the producers would
indicate their willingness and ability to produce through the
supply curve. However, there are differences in exactly how
the price is determined under each structure.

Body: Price
determination Revenue/
under PC Cost ($) MC Revenue/ Industry S
Firm Cost ($) P1

P AR= P
MR
D

0
Qe Output Q1 Q Q2 Output
0
Figure 1

Since a PC firm is a price taker who is too small to be able to


influence the market price, it can sell whatever output at a
constant price, i.e. their demand curve is horizontal or perfectly
elastic. Thus, the AR which is the demand is constant and is
equal to the price. Since it charges the same price for an
additional unit of good, its MR is also constant and equal to its
P and AR.
9
Thinking Process Content / Analysis / Evaluation

Referring to the figure above, from the market demand and


supply, the equilibrium price, P, and output, Q are obtained.
The PC firm, being a price taker, will set the same price as the
market. The equilibrium or profit maximizing output will be
where MR = MC at Qe. If the market price is at P1, there will be
a surplus as quantity supplied Q2 is more than the quantity
demanded Q1, indicating that the prevailing price is too high.
The surplus will thus create a downward pressure on price
causing the price to fall to the equilibrium price P. Conversely,
if the market price is too low, a shortage will occur and put
pressure on the price to increase till P. Thus, in a perfect
market, firms are price takers and price is determined by the
interaction of market demand and market supply.
Body: Price For firms under imperfect market, a monopoly would be used
determination to illustrate price determination. A monopolist is defined as a
under monopoly single supplier of a good or service for which there are no
close substitutes as the barriers to entry are high. For the
monopoly, the firm is also the market. Its large market power
makes it a price setter with the ability to set its own price.
However, they are still subjected to the law of demand,
meaning that if they want to sell more goods, they would need
to lower the prices of their goods. Thus, the demand or AR
curve is downward sloping and the MR curve lies below the
AR curve. Thus unlike a PC firm where AR=MR, under
imperfect market, AR>MR.

Revenue/Cost
MC (SS)
B
C
Pm Figure 2
Ppc A

D
E
AR (DD)
MR
0 Output
Qm Qpc
The monopolist determines its profit maximising output
according to the MC=MR rule. As shown in the above diagram,
the profit maximizing or equilibrium output will be where its MC
cuts its MR at point D. Thus, the monopoly is seen to be
producing output Qm. Its price will be Pm as indicated by its
AR (DD) curve. Since it decides on the output, the price is
determined by the position of the demand curve, that is,
the consumer decides.
Body: Highlight the Using the framework of a monopoly and PC market, the
differences of price differences in price determination can be highlighted. In a
determination perfect market, the market demand and supply will determine
between PC and the price and the PC firm, being a price taker will sell at the
monopoly market equilibrium price. Conversely, under imperfect market
10
Thinking Process Content / Analysis / Evaluation
like a monopoly, it has the ability to set price. Hence the
equilibrium output is first determined using the MC=MR
condition before getting the price from the consumers‟ demand
curve.

Conclusion In conclusion, the type of competition present will affect how


prices are determined. Besides a monopoly, a MC firm also
falls under imperfect market. Its pricing decision is similar to
that of a monopoly as it has some market power over its
product which is differentiated from the rest. However, no
industry price exists for MC as the firms sell differentiated
goods. Oligopoly, which falls under imperfect market as well,
has various ways in which individual firms determine their
prices. The firms could be engaged in price collusion or price
leadership.

Level Descriptor Marks


L3 Price determination in both perfect and imperfect competition 7-10
are well explained with a clear attempt to compare and contrast
to obtain top-end marks.
L2 Price determination in both perfect and imperfect competition 5-6
not well explained with some lapses e.g. confusion of firm and
industry and no clear compare and contrast framework.

L1 For an answer that shows weak and incomplete understanding 1-4


of the concepts. Focused on output rather than determination
of price.

11
(1) Analysis of Question (1-2 mins)

(b) Discuss whether the conduct of firms that operate in an imperfect market is
necessarily harmful to society. [15]

Command word
Discuss - requires
you to Key word Key word Key word
- use economic Key word Harmful – Conduct – to Imperfect
concepts to Necessarily need to examine the market – to
explain the – need to examine behaviour of examine the
impact on society examine the firms under impact on
of imperfect situations negative imperfect society of
market firms „ where effects of market MC,
behaviour imperfect imperfectly monopoly
- evaluate the market market and oligopoly
impact on society firms‟ firms‟ firms‟
of firms‟ behaviour behaviour
behaviour might be
beneficial

(2) Schematic Plan


Firms in imperfect
market

Harmful Beneficial
- not AE and PE - ability to reap EOS
- x-inefficiency - ability to do R & D
- inequity - can contribute to employment
of economy and increase
export revenue

(3) Write out full essay (20 mins)


Thinking Process Essay
Introduction As discussed above, imperfect market firms have some
degree of market power and are thus able to set prices. As a
result, firms under the MC, oligopoly and monopoly market
structures might be harmful to society, especially on the
consumers. However, there are also situations where such
firms bring benefits to the society and consumers.
Body: inability to Allocative efficiency is achieved when the society‟s welfare is
achieve AE maximised and it occurs when P=MC. It is achieved under PC
as the firm charges at P=MC. However, with reference to figure
2 from part (a), a monopolist can restrict output and charge a
higher price where P > MC, implying that the consumers value
an additional unit of the good produced more than the MC
incurred, thus being allocative inefficient. Allocative inefficient
implies inadequate amount of the goods is being produced.
Thus, consumer surplus is reduced compared to when the
good is produced in a perfectly competitive market. At the
same time, there is a deadweight loss of area CAD
12
Thinking Process Essay
experienced by the society, showing that while the producers
gain greater profits, it is not sufficient to compensate the lost
welfare by the consumers. A MC or oligopolistic firm with
market power is also able to do the same thing. Moreover, if
oligopolists act collusively and jointly maximise industry profits,
they will in effect be acting together as a monopoly. In such
cases, the harmful effects to society experienced under
monopoly will be experienced under oligopoly. Thus the
conduct of such firms would be harmful to society.

Body: inability to In addition, because of a lack of competition, the monopoly


achieve PE and x- may be complacent and not use the most cost efficient method
inefficiency of production. Thus, it may not be productive efficient as well,
as being PE requires the firm to be producing on the LRAC.
The monopoly may also not have any incentive to reduce its
average cost as it could merely charge consumers a higher
price to cover its higher AC. In addition, with barriers to entry,
the monopolist can continue to earn supernormal profits in the
long run as it can continue to keep its price above its AC. Thus,
consumers end up paying a higher price. This is made worse
by the phenomenon of X-inefficiency whereby the monopoly
produces at a point above the LRAC curve. A MC and
oligopolistic firm would also be producing at excess capacity in
the short run, thus not achieving PE.
Body: inequity A monopolist and oligopolistic firm can earn supernormal profits
even in the long run due to the substantial barriers to entry.
Thus, income is being redistributed from the consumers (the
many) to the firms (the few). This will increase income inequity
in the economy and is thus harmful to society.
Body: ability to reap Though the conduct of firms under imperfect market might
EOS seem to be harmful to society, there are circumstances where
they may be beneficial to society. The monopolist can actually
charge a lower price and produce a higher output compared to
the PC industry when it reaps substantial internal economies of
scale. As the PC industry comprises many small firms, each
firm is not able to reap substantial EOS. On the other hand, the
monopolistic firm which produces on a large scale is able to
reap internal EOS, which enables its MC curve to be lowered
and shift from MC to MC1 as shown in figure 3.
Price per unit
MC

MC1 Figure 3
Pm
PC
P1

MR AR
0 QmQc Q1 Output

13
Thinking Process Essay
As a result, the monopolist is able to charge a lower price and
output at 0P1 and 0Q1 respectively compared to that of the PC
industry where the price is at 0PC and output, 0Qc. Hence, if the
monopolist is willing to pass the cost savings to the consumers,
the consumers may actually enjoy goods of lower price and
hence a monopoly need not necessarily be harmful to society.

The benefit arising from EOS is even more prominent in the


case of natural monopolies. A natural monopoly occurs in an
industry where there is substantial cost savings due to large
scale production, e.g. the utilities industry. As a result, the
LRAC of a natural monopoly is falling even when the entire
market demand is satisfied. Thus it is more efficient for one firm
to produce for the entire market. However, given that a natural
monopoly has substantial market power even in the long run, it
is possible that it does not pass the cost savings to the
consumers. Hence, the government may want to regulate the
monopoly so that it would not exploit the consumers but instead
would pass the cost savings to the consumers.

Body: ability to do A monopoly or oligopolistic firm has the financial ability and
R&D incentive to innovate and engage in R&D due to their
supernormal profits. Besides, such market structures may be
more dynamic efficient than PC. They have the financial
resources to engage in R&D which may help to lower the price
and raise the quality of its good. Moreover, in trying to maintain
their barriers to entry, they may have the incentive to innovate
and improve the quality of its product. On the contrary, PC
lacks resources and the incentive for innovation and hence
consumers may not always benefit. The incentive to innovate
by the monopolist is stronger especially in a contestable
market. This happens when the market has low cost of entry
and exit which increases potential competition in the market by
“hit and run” producers. Thus, the monopolist must be on its
toes and constantly innovate to keep its prices low and quality
high so as to prevent new firms from usurping their monopoly
position. Thus, consumers are not necessarily worse off.

Body: Macro benefits In the increasingly globalised world, the presence of foreign
competitors means that monopoly and oligopolistic firms face
stiff competition. This can help to minimise the risk of
exploitation by the domestic firms. Such firms also help to
promote the macroeconomic aims of employment, economic
growth and BOP as large firms such as monopoly and
oligopolistic firms offer greater stability and compete more
effectively in the global market, thus increasing the export
revenue. This would benefit the society, especially consumers
as their SOL can increase with a rise in employment and
national income.

Body: Product variety Firms under MC structure can also be beneficial to society as
and advertising they offer consumers with a wide product variety in terms of
style, quality, services and design to suit their differing tastes
14
Thinking Process Essay
and preference, increasing consumer welfare. The large variety
of restaurants and retail shops available certainly help to
increase the choices available to consumers. Such firms also
engage in non-price competition through advertising and
product development. This in turn creates jobs for the
workers in industries like advertising companies,
broadcasting stations and publishers etc.
Conclusion In conclusion, it can be seen that the conduct of firms that
operate under imperfect market is not necessarily harmful to
society. Though they tend to be allocative and productive
inefficient, there are possible circumstances where they benefit
the society in terms of lower prices due to EOS, more
innovations and achieving the macro aims. Moreover,
industries which involve a high start-up cost like utilities might
be more suited to a natural monopoly structure. The
government can do its part in minimising the harmful effects by
regulating the big firms to ensure that they do not engage in
unfair practices which may restrict competition. Government
can regulate them using policies such as taxation and pricing
policy. The important thing is that the regulation should
consider carefully the costs and benefits so as to lead to a
more efficient allocation of resources and higher welfare for the
society.

Level Descriptor Marks


L3 Clear discussion of both beneficial and harmful effects across 9-11
the 3 market structures under imperfect competition. (2 harmful
and 2 beneficial effects)
L2 Explanation of both beneficial and harmful effects but lacks 6-8
development OR well-developed answer on either the
beneficial or harmful effects only. (2 harmful and 1 beneficial
effects or 1 harmful and 2 beneficial effects)
L1 For an answer that shows some knowledge of conduct of firms 1-5
under imperfect competition. Mere listing of effects.
E2 For a conclusion with sound justification and evaluation based 3-4
on analysis of circumstances of the various outcomes possible.
E1 For an unexplained conclusion on the effects of the firms‟ 1-2
conduct.

15
3 The objectives of government microeconomic policies are to achieve efficiency
in resource allocation and to promote equity in income and wealth distribution.

Assess the various policy measures that the Singapore government currently
adopts to achieve the above microeconomic objectives. [25]

(1) Analysis of Question (to be done in 1-2 mins)


Assess the various policy measures that the Singapore government currently
adopts to achieve the above microeconomic objectives. [25]

Command
word - Assess
requires you to Key word:
Key word:
apply and Policy – Key word: Key word:
evaluate Singapore –
Need to Microeconomic Currently -
Need to
economic state, objectives – Need Note to look at
policies to examine the
explain and to explain how the the existing
achieve the policies in the
evaluate the govt can bring policies
micro aims Singapore
policies. about a more
context
(SEE) efficient resource
allocation and a
more equal
income distribution

Question requirement
This question requires you to clearly explain and evaluate the policies that the
Singapore government use to achieve its micro aims of efficiency and equity in
income distribution.

(2) Schematic Plan (to be done in 2-3 mins)


Micro aims

Efficiency (allocative efficiency) Equity in income distribution

Market failure

Externalities Market power


Progressive tax Subsidies to
lower income,
Labour schemes
Positive Negative Anti-trust
laws
Eg: healthcare ciagarettes liberalisation
Education congestion

Subsidies Taxes
Rules & regulation, campaigns

16
(3) Write out the full essay

Thinking Process Content / Analysis / Evaluation


Introduction: The government has two micro objectives of achieving efficiency in
Define micro resource allocation and equity in income distribution. Efficiency in
objectives of resource allocation would mean achieving allocative efficiency
the where no redistribution of resources would make someone better off
government without making someone worse off. When AE is achieved, society‟s
welfare is maximised and the condition to achieve this is when
price=marginal cost. Equity in income distribution is to ensure a
more equal distribution of income and wealth and it is usually
measured by the Gini coefficient where the closer the value is to 1,
the higher the income inequality in the country.
Introduction: Without government intervention, the price mechanism will allocate
- Market failure resources to their various uses, where the interaction of demand and
- Trade off supply will determine the price. The price mechanism can only
between achieve efficiency if the following assumptions hold: perfect
efficiency and information, perfect mobility of factors, absence of externalities,
equity markets for all goods and perfect markets. However, in reality, such
assumptions might not hold, leading to market failure where the price
mechanism fails to allocate resources efficiently. As such, the market
does not provide the optimal amount of a particular good. Thus there
is a need for the government to intervene and help bring about
efficiency. The price mechanism allocates resources based on the
dollar vote where goods and services are produced for consumers
with the ability to pay for them, thus equity is not ensured in the
economy. Hence, the government might need to intervene to reduce
the income gap as a widening income gap may lead to social
discontentment which may generate disturbances to the production
processes, creating an avenue where resources are inefficiently
employed if such disturbances were to occur. This essay will look at
the market failure caused by externalities in Singapore and also how
the government brings about a more equal distribution of income.
Body: An optimum resource allocation cannot be achieved with the existence
Negative of externalities. Externalities arise whenever the production or
externalities – consumption of a good has spill-over effects beyond the producers
congestion and and consumers involved in the market and these spill-over effects are
pollution not compensated. There are positive and negative externalities. In
Singapore, a number of policies are implemented in view of market
failure that occurs due to external costs arising from car use,
congestion and pollution and external benefits in the consumption of
education and healthcare. Policies intervention in such markets are
intended to bring the level of output to the socially optimum level,
thereby achieving efficient allocation of resources, which is one of the
microeconomic goals of the government.
Costs/Benefits ($) Welfare Loss MSC
MPC

MEC
Ps

Pp

MPB = MSB
=D
0 Quantity 17
Qs Qp
Thinking Process Content / Analysis / Evaluation

Traffic congestion is an example of market failure in Singapore.


Referring to the figure above, left on its own, market equilibrium will be
at Qp where MPB cuts MPC. Only private benefits and private costs
are considered in the decision making process. As a person drives, he
would only take into consideration his private cost e.g. fuel costs but
not the external cost he imposes on others e.g. loss of productive
hours and rise in pollution level. Thus, resulting in the divergence
of MSC and MPC, leading to an over-usage of cars at the market
equilibrium since the socially optimum number of cars on the road
should be at Qs where MSC = MSB i.e. the additional cost of
having an extra car on the road is equal to the additional benefit
of the last unit of car on the road. Society‟s welfare can be
increased by reducing the usage of cars. The shaded area of welfare
loss results in market failure.
Hence the Singapore government has taken drastic steps to get car
drivers to internalise the external cost. The main direction taken is
to tax car drivers an amount that is calculated to be the external cost
imposed on society. This causes the marginal private cost to converge
to the marginal social cost and it will eliminate over-usage, causing the
amount of pollution and traffic congestion to be at an optimal level.
These taxes came in the form of road tax, COEs and ERP.
For the ERP system, the government identified stretches of roads
which were frequently congested and erected electronic gantries.
Every car that passes through these gantries at fixed timings would
have a fee deducted from their cash cards. Hence the tax paid by car
users along these roads are higher as the external cost they impose
on society is higher. These roads tend to be the ones that lead to the
central business district where any impact of congestion on
productivity is very high.
This measure has proven effective as it has reduced congestion in
those areas. It has distributed traffic to other areas. It is also flexible
as the cost of using the selected roads can be easily increased if the
traffic situation does not improve. This allows the system to get around
the difficulty of estimating the external cost imposed. Wrong
estimations can be easily rectified by adjusting the ERP charge.
Cameras have been installed to monitor the traffic situation. Once the
traffic builds up to cause congestion over a sustained period, the
charges will be reviewed. To a great extent it has been successful as
countries like London have modelled after the ERP system to combat
congestion
However, it must also be noted that demand for travel into the city
area tends to be price inelastic. Given that these drivers earn a
relatively higher income, the ERP charge is a small proportion of their
income. Moreover they may view driving to work in the city area as
essential. Hence the rise in price will lead to a less than proportionate
decrease in quantity demanded. Hence a two prong approach is
needed to alleviate the congestion problem.
The Singapore government has done this by engaging in road
expansion through building more multi-tiered highways. More
importantly, they have attempted to improve on the public
18
Thinking Process Content / Analysis / Evaluation
transportation system to provide a good substitute to driving. This can
be seen by the building of the Circle line, allowing commuters to by-
pass the city area in their travel. They are also trying to ensure
frequent bus services where satellite tracking allows commuters to
estimate the arrival of the bus. Once a good public transportation is
available, commuters will be more price sensitive to rises in the ERP
charges and make the switch over. However it must be noted that
scarce resources will have to be diverted from other productive areas
to improve on the public transportation system.
Body: Education is deemed as a merit good as the government considers it
Positive to be socially desirable and it generates significant external benefits
externalities – for society. A person who has received an education not only benefits
education from having more knowledge and skills but has higher productivity and
is able to command a relatively higher wage for his contributions at the
market place. At the same time, the rest of society benefits as having
a more efficient workforce usually attract foreign investors to invest in
a country and the economy is thus able to expand further, generating
higher levels of income for the country. However, a person who is
undertaking education will take into consideration only the benefits he
can gain, or the marginal private benefit (MPB) of consuming
education, and ignore the benefits that the rest of society, or the
marginal external benefit (MEB) can enjoy. In other words, there is a
divergence between the marginal social benefit (MSB) and the MPB of
consuming education, leading to under consumption of it and this can
lead to market failure.
To encourage higher levels of consumption, the Singapore
government can give a subsidy equivalent to the MEB, thereby
encouraging consumers to raise their consumption of education. For
example, local students are paying fairly low monthly fees as they
have been enjoying heavy subsidies in most publicly-run schools. In
addition, they have received extra funding through schemes like the
Education Endowment Scheme that allows them to pay for enrichment
and co-curricular activities organized by the schools.
However, it remains difficult to estimate the level of marginal external
benefits and therefore the correct level of subsidies to be granted
cannot be correctly calculated. Too much subsidies results in
overconsumption of education and hence wastage of resources, while
too little subsidies will result in insufficient level of education being
consumed.
The government also came up with legislations regarding compulsory
primary education in 2003. The government then subsidizes
educational institutions by bearing the full capital costs of all tertiary
institutions and the bulk of capital costs of most educational
institutions. All these will enable more education to be provided at
lower fees, which will benefit both the poor and the rich. However, in
general, Singaporeans value education as a ticket to a better life.
Even without the legislation, Singaporeans will want to pursue more
and better quality education.

Body: Equity in The other micro aim of the government is to adjust the distribution of
income income and wealth to improve equity based on what it sees as fair and
distribution just. The most widely used method for measuring inequality of income
(Causes of is the Gini Coefficient. It ranges from 1(perfect inequality) to zero
19
Thinking Process Content / Analysis / Evaluation
widening (perfect equality). Singapore has witnessed rapid growth over the last
income gap in 40 years. However, this rapid tide of growth and material wealth
Singapore) favoured some far more than the others. Between 2005-2007, the
average income for families in the bottom decile of the income
distribution rose just 3-4% while that of the topmost decile grew by 6-
11%. The Gini coefficient rose from 0.467 in 1999 to 0.485 in 2007.
This shows rising income inequality.
Globalization and the largely open nature of the Singapore economy
have played a key part in creating this inequity. Free trade and foreign
direct investment might result in lower wages or unemployment for low
skills workers in developed economies like Singapore. For example,
when a local manufacturer decides to close down the factory in
Singapore and relocate to China, the redundant workers increase the
supply of labour in Singapore and drives down domestic wages. On
the other hand, exports industries that sell to the global market would
demand for more highly skilled workers, contributing to higher wages
for these workers. Moreover, the government‟s manpower policies
towards attracting foreign semi- and unskilled workers (which depress
the wage of lower-skilled workers) and foreign talent (which helps to
raise the wage of the local skilled workers to match the foreign
expatriates) have further intensified the problem in recent years.
Recent adaptations to the tax structure (shift from direct to indirect tax
which makes the tax system less progressive) have also aggravated
the problem as the raising of the GST from 5 to 7% would penalise the
poor more as it is regressive in nature. The lowering of the income tax
to 20% would reward the rich more than the poor, thus contributing to
a widening income gap.
Income inequality can have profound ill effects, especially for a
service-oriented economy and multi-religious society like Singapore. It
can push low income families into poverty. Racial bias of income
inequality in Singapore could trigger social unrest. Increased sense of
vulnerability may drive up mortality rates and crime. Thus inequity can
rob Singapore of the very factors – law and order and efficiency, which
placed Singapore at the helm of the globalized world. Thus there is a
need for the government to intervene and improve equity.
The government has relied mainly on taxation, subsidies and labour
schemes to mitigate the problem. Singapore adopts a progressive tax
system where the rich would be taxed more heavily than the poor.
This would help to narrow the income gap. However, personal income
tax rates have fallen over the years. In 2006, the tax rate for top-
income earners (i.e. gross annual income > $320,000) is 21% while in
2007/8, it fell to 20%. Corporate tax rate has also fallen. From 2008
onwards, a company is taxed at a flat rate of 18% on its chargeable
income. This will be reduced to 17% from 2010. Although the tax rates
on direct taxes (i.e. personal and corporate) have fallen, GST (an
indirect tax) has been increasing. The tax system has been shifting
away from reliance on direct taxes towards indirect taxes in order to
keep Singapore‟s tax system internationally competitive. With the
increase in GST, income taxes could be reduced. The objective is to
promote greater work incentives and to keep Singapore‟s personal
and corporate tax rates internationally competitive. However, this shift
in tax structure will make it more regressive hence penalizing the
lower-income households. The government has thus tried to minimize
20
Thinking Process Content / Analysis / Evaluation
the negative impact on income distribution by giving more subsidies to
the lower income households (e.g. increasing the subsidies for
conservancy charges, workfare supplements for lower income
earners)
The government has also constantly upgraded the skills of the
workers so as to prevent rising structural unemployment and narrow
the widening income gap. To minimise the economic problems due
to greater volatility, Singapore has also put up more social safety net
to help workers who are affected by the structural changes and
economic fluctuations. Thus, policies such as the Workfare Package
are designed to help to increase the income of lower-income
workers and Job Credit Scheme are aimed to encourage firms to
send their workers for training rather than to retrench their workers
during a economic recession. Attempts to establish a robust security
system that addresses the longevity issue has already begun with
the introduction of the CPF Life scheme. The WIS (Workfare Income
Supplement) also plays its part in addressing income inequality in
the low-wage segment. In general, all these schemes and subsidies
serve to increase the income of the lower income group and improve
their living standards. These policies supported by a multi-tier social
security system could hopefully turn the tide on inequality and
ensure that the gains from economic growth are more widely
shared. However, it must be noted that an opportunity cost is
incurred when the government uses funds for such purposes as less
funds would be available for other sectors like healthcare and
national defence. Conversely, if these funds come mainly from the
taxes from the rich, the income inequality situation can be improved.
Conclusion In conclusion, the government has implemented a range of policies to
try to achieve the micro objectives of efficiency in resource allocation
and equity in income distribution. In terms of market failure due to
externalities, the current policies have been fairly successful as our
ERP system has been modelled by other countries and the continued
upgrading of our public transportation system would serve to get more
people to switch from cars to public transport. In terms of education,
our literacy rate of over 90% is commendable and our enrolment rate
in tertiary education has been fairly decent at about 25%. Singapore is
also well known for a well-educated and quality workforce which is a
major pulling factor for FDIs coming in. Perhaps the government can
explore the option of lesser intervention to influence the allocation of
resources by the price mechanism in this market. e.g. allow more
private schools to set up and offer a greater range and varieties of
educational packages. In addition, with globalization and the transition
to higher value-added industries, increasingly there is a stronger
demand for workers with higher qualifications (e.g. PhD). However,
these workers are lacking in Singapore. Hence, there might be a need
to look into encouraging students to pursue further studies. In terms of
income equity, there has been a slight increase in income inequality in
Singapore. There might be a need for the government to look into
providing more subsidies and schemes to help the lower skilled and
lower income, especially as Singapore is increasingly plugged into the
globalised world.

21
Level Descriptor Marks
L3 For an answer that uses analysis to underpin the discussion of 15-21
the micro problems in Singapore and a good discussion of the
policies used by the government to achieve the micro aims. (2
policies for each problem or 3 policies for 1 problem and 1
policy for the other.)
L2 For an answer that gives an explanation of the policies adopted 10-14
by the Singapore government.
(1 policy for each problem explained very well or 2 policies for
1 problem and 1 policy for the other problem)
L1 For an answer that shows descriptive knowledge of policies or 1-9
an unexplained list of one or more examples of market failure
and income inequality in Singapore.
E2 For a discussion justifying the conclusions drawn. 3-4
E1 For an unexplained evaluative statement(s) without 1-2
justification.

22
4 (a) Explain the effects of an increase in investment by firms on output and
prices in an economy. [10]

(b) Discuss how the Singapore government influences private investment to


ensure sustained economic growth. [15]

(1) Analysis of Question

(a) Explain the effects of an increase in investment by firms on output and


prices in an economy.

Command word: Key words:


Explain: requires you
Effects – short term and long term
to establish the causal
links between a rise in Output
investment and output Prices
and prices.

Question requirement
This question requires you to clearly explain how a rise in investment affect prices
and output level in an economy.

(2) Schematic Plan

Effect of an increase in Investment


oglobalization

Short run Long run

- ↑ output thru the k process - I increases LR


- Effect on output and price depends on size of k productive capacity
and state of economy  ↑AS  real o/p ↑,
- If economy at < full employment: real o/p ↑, P P ↓ in the long run
relatively stable
- If economy at full employment: real o/p un∆, P ↑

Conclusion

1
(3) Write out the full essay

Thinking Process Essay (content, analysis and evaluation)


Introduction: Define investment – planned spending on capital goods such as plants,
Define key concepts machines and raw materials. It can be divided into autonomous and
induced investment. Autonomous investment is determined
independently of the level of aggregate demand in the economy. It is
determined by non-income factors such as technological progress,
interest rate, business expectations, government policy etc. induced
investment, on the other hand, is determined by the rising national
income

Explain the effects An increase in autonomous investment by firms will bring about a
on prices and multiplied increase in national income which is known as the multiplier
output using the process.
multiplier concept.
The increase in output is a multiple of the initial change in aggregate
expenditures because consumption is stimulated by the circular flow of
income and output. Once aggregate expenditures stimulate the circular
flow, total production continues to increase as consumption, production,
and income increase in each subsequent round of the circular flow.

Multiplier process: Suppose the economy is at an equilibrium position and then autonomous
State the net investment rises by S1m due to an increase in the output and sale of
assumptions one machine priced at $1m. The people who produced this machine
receive an extra S1m in income. Thus, income rises by $1m in the first
round. The people who produced the machine will spend some of the
increase in their income and withdraw some of it in the form of saving,
taxes and imports. Because one person‘s expenditure is another‘s
income, income will rise again. Thus if the MPC is ¾ , the group that
produced the machine will spend $750,000 on goods and services and
will withdraw $250,000. The $750,000 spent becomes income for the
people who produced the $750,000 worth of consumer goods. The
process will not stop here. The people who just receive an increase in
income of $750,000 will spend some and withdraw some. The amount
they spend becomes income for others. This process only stops when
the total withdrawals is equal to the initial increase in expenditure. The
total increase in national income will be increase in I x k, where k = 1/1-
MPC. K will be 4 in this case. Hence total increase in Y = $4m.

Thus an increase in investment will lead to a rise in national income, but


whether real output or prices will rise depends on if the economy is at
below or at full employment level.

An increase in investment will cause aggregate demand to shift to the


right. If the economy is producing below full employment, since the
economy has excess capacity, it could utilize its unemployed resources
to produce more goods to meet the increase AD. Hence the price level

2
Thinking Process Essay (content, analysis and evaluation)
remains the same. This is reflected by the AD curve falling on the
horizontal range of the aggregate supply curve. Thus, its real output
increases. However, if the economy is already producing near or at full
employment, it means there will be more intense competition for the
scarce resources. This will push up the general price level. This is
reflected by the aggregate demand curve moving up to the vertical range
of the AS curve which indicates that the resources in the economy are
fully utilized. Hence, any increase in aggregate demand will only lead to
a rise in prices, causing a demand pull inflation. Thus there is no
increase in real output.

The multiplier indicates how national income and price level increases as
a result of an increase in investment – a component of aggregate
demand. The ultimate increase in national output will be greater than the
initial increase in investment. How large will the increase in nominal
income depends on the size of multiplier. Which is directly dependent on
the MPC. The larger the MPC the larger the size of k and hence the
greater the impact an increase in investment has on national income

Long run effect Besides increasing output in the short run, investment can also expand
the productive capacity of the economy in the long run. Higher
investment leads to higher productivity and thus increases AS and hence
long term economic growth of output as well as a fall in GPL.

Conclusion In the SR the effect on output and prices due to an increase in


investment is uncertain as it depends on the size of k and the state of the
economy. In the long run however, the impact tends to be more certain.
Given the positive impact higher investment have on the economy, it is
therefore important for the government to ensure there is sustained
investment in an economy through the implementation of various
policies.

Level Descriptors Marks


L3 Thorough analysis of effects on output and prices bringing in the 7-10
relevant assumptions
Long term effects on the economy
L2 A good attempt at explaining the effects on output and prices but 5-6
with limited analysis on the assumptions on size of k and state of
the economy

L1 Smattering of points with no economic concepts at all or with 1-4


many conceptual errors.
Explanation shows little understanding of effects on output and
prices of an economy

3
(b) Discuss how the Singapore government influences private investment to
ensure sustained economic growth. [15]

(1) Analysis of Question

(b) Discuss how the Singapore government influences private investment to ensure
sustained economic growth. [15]

Command word: Context: Key concept:


Discuss Apply to Singapore context Sustained economic
growth – short term and
long term growth

Question requirement
This question requires candidates to clearly explain and evaluate at least 3
government policies that influence private investment to ensure sustained (long
term) economic growth.

(2) Schematic Plan

Definition of sustained economic growth

Investment decisions hinges on interest rates and


expected rate of returns.

Interest rates beyond the control of Singapore govt,


hence govt policies target at increasing expected
rates of returns

Policies affecting expected rate of returns – various


fiscal measure – tax incentives, infrastructure, etc

conclusion

4
(3) Write out the full essay

Thinking Process Essay (content, analysis and evaluation)


Introduction: Economic growth is the process of increasing the economy‘s ability to
Define key concepts produce goods and services. It is seen as an outward shift in the
production possibility frontier or a rightward shifts in the aggregate
supply curve. It is measured by changes in real GNP/GDP. With this
increase in real income, more wants of the people can be satisfied and
hence an improvement to living standard. In order to have a sustained
increase in real GNP in the medium or long term, sufficient economic
growth is necessary. To have sufficient economic growth, the economy
has to grow fast enough so that more jobs are created for the people in
the country and unemployment does not become a problem for the
economy.

Explain how Investment decisions depend on the expected rate of returns and cost
government can of borrowing (which is interest rates). Firms will invest up to the point
influence private where the expected rate of returns is equal to the cost of borrowing
investment which is interest rate. For countries such as Singapore which is small
and very open, its interest rate is tied to the international interest rate
and hence its government cannot use interest rate policy to stimulate
investment to bring about a higher growth. Instead what the
government uses are various fiscal policies to increase the expected
rates of returns from investment. To ensure that economic growth is a
sustained one, the types of investment Singapore attracts is
important. Singapore has done well in attracting high value-added
investments in manufacturing, services and R&D with the use of
various policies.

i) Pro-Business Environment (Infrastructure, govt policy)


 The availability of quality infrastructure such as transportation,
electricity, water and telecommunications, is an important
determinant of investment.
 This will increase connectivity, boost efficiency and productivity →
cost of production ↓→ expected profits ↑.
 As announced in Budget 2009, our corporate tax will also be
reduced to 17% from 2010, making Singapore the country with one
of the lowest corporate tax rates, so as to provide big foreign firms
(MNCs) with more incentives to  investments in Singapore.

Governments around the world have dropped their corporate tax


rates over the last five years, including those in Eastern Europe
and even France.

5
Thinking Process Essay (content, analysis and evaluation)
To keep Singapore as an attractive business location, Singapore
cut its corporate tax rate from 18 to 17 per cent for the year of
assessment 2010 to boost competitiveness and lure investment to
spur job creation. According to the government, this would save
businesses up to between $400 and $500 million a year in the
medium term. The cut in corporate tax is a signal of the
government's continued and future commitment to being the best
hub for enterprises, small and large, from all over the world. The
tax cut will narrow Singapore's gap with Hong Kong as the
government aims to attract investment in services and
manufacturing industries.

With the announcement for a lower tax level made in early 2009,
Singapore has shaved nine percentage points off the corporate tax
rate since 2000. The government reduced rates by 2 percentage
points to 18 per cent in 2007. Hong Kong last year lowered its
company tax rate by 1 percentage point to 16.5 per cent. This is
the second time in three years Singapore has cut corporate tax
rates to boost competitiveness and lure investment to spur job
creation amid the island's deepest recession since independence.

Evaluation
 direct and tangible
 depends on firms ‗ expectation of future income / returns
 needs to ensure that our corporate tax rates remain competitive
against that of our neighbouring countries e.g. HK.

ii) Availability of Factors of Production (Eg. Labour, raw


materials, capital)
The availability of skilled human capital will also attract investment to
Singapore, particularly for sectors that are relatively intensive in the
use of skilled labour.
 In Singapore, programmes such as Skills Programme for
Upgrading and Resilience (SPUR) which are run by Workforce
Development Agency (WDA) and Employment & Employability
Institute (e2i) to upgrade workers‘ skills. This is to ensure that our
workers are constantly equipped with the necessary skills for the
knowledge-based economy.
 These policies ensure ↑ labour productivity can match with wage
increases → cost of production ↓→ investments ↑.

6
Thinking Process Essay (content, analysis and evaluation)
iii) Economic Stability
E.g. price stability, sustained economic growth
During times of low inflation, there is price stability → expectations of
profits  because wage costs tend to lag behind prices of goods and
services → incentive for profit-maximising producers to  investment.
Conversely, severe inflation increases uncertainty and risk of future
rates of return on investment → firms find it difficult to make accurate
projections on future prices, costs and profits → investment ↓.

Evaluation
Economic stability involves long term planning. Long term planning
involves a lot of uncertainties which are unpredictable such as world
economic and political conditions may change

Conclusion Investment is undeniably a necessary condition to promote sustained


economic growth of a country. The success of many Asian economies
including Singapore further highlights the importance of the
government in playing an active role in increasing investment to
promote economic growth. Increasingly, with globalization, the
competition for investment has intensified further thus implying
pressing need for government to implement effective policies in
promoting investment in the country.

Level Descriptor marks


Level 3  Thorough analysis of at least 3 policies affecting I in 9-11
Singapore.
 Good use of examples to show application and understanding
of the Singapore economy.
 Excellent application of relevant economic concepts with
little/no errors.
Level 2  A fair attempt at explaining the factors influencing I in 6-8
Singapore.
 Good application of economic concepts.
 Some reference to the Singapore‘s economy with use of
some examples.
Level 1  Smattering of points with no economic concepts at all or with 1-5
many conceptual errors.
 Explanation shows little understanding of what influences the
level of investments and the nature of Singapore economy.
Evaluation
E2 A well developed judgment 3-4
E1 Making an undeveloped judgment on – effectives of policies, 1-2
types of investment to ensure sustained EG

7
5 A record level of retail spending in May has fanned fears of higher interest
rates.
Adapted from http://news.bbc.co.uk, 19 June 2008

(a) Would you support the implication of this statement that inflation is caused
by consumer spending? [10]

(b) Discuss how effective interest rates alone might be in controlling inflation if
the rate of inflation were to become unacceptably high. [15]

(1) Analysis of Question

(a) Would you support the implication of this statement that inflation is
caused by consumer spending? [10]

Command word: Key word: Key concept:


Would you support Which statement? Inflation – caused by
requires you to ‘record level of retail excessive consumer
- assess and evaluate spending’ spending?
theory Which imply there is
- consider: is theory
significant
excessive consumer spending
- consider the ‗it
depends‘ clause

(2) Schematic Plan (to be done in 3-4 mins)

Question requirement 1) Economic slowdown 2) liberalisation


This question requires you to clearly explain to what extent inflation is cause by consumer
spending

Define inflation and identify demand and cost push


inflation

Explain how higher retail spending may imply higher


consumer spending and hence causing inflation

Examine unstated assumptions – inflation need not


be caused by just consumer spending alone

Other causes of
inflation

Conclusion

8
(3) Write out the full essay

Thinking Process Essay (content, analysis and evaluation)


Introduction: Inflation is defined as a period of sustained, inordinate increase in the
Define key concepts general price level. There are mainly two types of inflation, demand pull
and cost push inflation. Demand pull inflation occurs when there is
excessive aggregate demand when the economy is near or at full
employment income level, while cost push inflation occurs when there is
an increase in cost of production leading to a left ward shift of the AS
curve.

Explain Effects on Growth in retail spending may be due to higher consumption  this
general price level brings about higher aggregate demand. This is particularly so if
due to a growth in consumption is a significant component of AD such as the case of the
retail spending US. This is shown by a rightward shift of the AD curve. If this occurs
when the economy is approaching full employment or when the economy
is at full employment level of income, this will pull prices up, causing
demand pull inflation. (hence ‗has fanned fears of higher interest rates‘)
This is because there are no more resources left to increase production
to meet the increase in AD. When AD increases, firms will attempt to
increase production to meet the increase in demand. Competition for
factors of production would increase and factor prices will also increase.
Demand pull inflation is characterized by shortages, low level of stocks,
long waiting lists and queues. In this situation, firms will be eager to
produce more as soon as they can. They may invest in extra capacity,
but there are some goods whose supply is inelastic. In the short run,
aggregate supply is likely to be inelastic because firms may not be able
to recruit staff easily or produce more given the existing production
capacity. With the higher cost of production, prices of goods and
services would increase. A new equilibrium is reached when there is
higher price but real income remains unchanged.

However, the preamble did not indicate the degree of significance of


consumption in an economy. As such, given the inadequate information,
inflation may be caused by other causes as well.

Demand-pull inflation can be caused by higher exports, higher


investment and higher government spending, not just by higher
consumer spending.

In addition, higher inflation can be due to rising cost of production


caused by lower exchange rate (which lead to more expensive imports),
higher wages, higher rentals and higher indirect taxes. (diagram to
illustrate) Faced with higher costs, firms increase their prices to
consumers to maintain profit margins. This shifts the AS curve to the left
and causes cost push inflation.

9
Thinking Process Essay (content, analysis and evaluation)
conclusion Thus, inflation may not be caused solely by higher consumption. There
are other possible causes as well. Since there is inadequate
information on the other causes, it is not possible to conclude that
inflation is caused by higher consumption alone.

Level Descriptors Marks


L3 Thorough analysis of how lack of sufficient data make it difficult 7-10
to pin point the exact cause of inflation
L2 Plus an attempt to point out other wise 6

A good attempt at explaining how record level of retail spending 5


imply higher consumer spending and hence higher demand pull
inflation

L1 Smattering of points with no economic concepts at all 1-4


Going into the 2 different types of inflation with no link to the
preamble and question. Explanation shows little understanding
of how record level of retail spending imply higher consumer
spending leading to inflation

10
(1) Analysis of Question

(b) Discuss how effective interest rates alone might be in controlling inflation if the
rate of inflation were to become unacceptably high. [15]

Key word: Key word:


Command word: Interest rates alone – is Could be above the
Discuss requires you to interest rate alone enough targeted inflation
select and apply economic to control inflation?– Need
concepts and principles (in
to identify the causes of
this case interest rates ) to
control inflation inflation to decide whether
i/r alone is enough

Question requirement
This question requires you to consider whether interest rates alone is enough to
control inflation

(2) Schematic Plan

The importance of controlling inflation

Explain how higher i/r control inflation

Limitations of i/r in controlling inflation

Is i/r enough to control inflation?

Other policies to control


inflation – depending on
causes

Conclusion

11
(3) Write out the full essay

Thinking Process Essay (content, analysis and evaluation)


Introduction: An unacceptable high rate of inflation brings about undesirable effects on
The importance of the internal and external sector of an economy. There is thus a need for
controlling inflation the government to implement appropriate measures to control such high
inflation.

Explain the use of One of them is through the use of money supply to increase interest
monetary policy to rates. According to Monetarist, money supply influence price level
control inflation directly and thus monetary policy is effective in solving inflation.

Internally, a higher interest rate would imply that the cost of borrowing is
increased and at the same expected rate of returns, previously profitable
investment projects will now appear less profitable. Hence investment all
.
In addition, the higher interest rates would discourage consumption as
consumers find their returns from savings increased. The opportunity
cost of current consumption has increased. Hence consumers would
rather save than spent. Hence consumption falls.

Together with a fall in C and I, AD fall. When the economy is at full


employment, a fall in AD results in a fall in general price level.

Externally, higher interest rates will lead to greater capital inflow and less
capital outflow. This will lead to an increase in the demand for domestic
currency and fall in supply of domestic currency. The external value of
the domestic currency will appreciate. As a result of the appreciation,
price of imports will fall in domestic currency and this helps to reduce
inflation, especially if the country imports extensively consumer and
capital goods. The fall in price of imported capital goods also lower the
cost of production and leads to lower cost push inflation. As the price of
imported consumer goods fall, consumers will also not be pressured to
demand for higher wages. This help to keep the wage cost low and thus
further help in reducing cost push inflation.

In addition, the higher exchange rate will cause the price of exports to
rise in foreign currency. This will lead to a fall in quantity demanded of
exports. Assuming elastic demand for exports, the value of exports will
fall and thus the aggregate demand will fall. This will help to reduce
demand-pull inflation assuming elastic demand for imports and exports.
If demand for imports and exports is inelastic, a rise in exchange rate will
improve the balance of trade and increase the aggregate demand, thus
worsening demand-pull inflation.

The extent interest rate alone is effective in reducing inflation depends

12
Thinking Process Essay (content, analysis and evaluation)
Evaluation on the interest elasticity of consumption and investment. If consumption
and investment are interest inelastic (possibly due to optimism as
Whether Interest consumer and investors will continue to borrow despite the high interest
rate alone is rate), then higher interest rates alone may not be enough in controlling
enough
demand-pull inflation. However, despite the possibly weak interest rate
effect, many central banks still resort to MP to influence i/r and price level
as i/r rates can be easily and quickly changed to influence aggregate
expenditure, thus reducing the problem of time lag.

Furthermore, it depends on whether the Marshall-Lerner condition holds


true. Theoretically, the condition holds true only in the long run.

The nature of the economy also determines whether interest rates alone
is enough to reduce inflation. For countries such as Singapore who has
no control over interest rates, interest rate fails completely as a policy to
control inflation.

Monetary policy which raises i/r may also be unpopular as it raises the
debt burden of home owners who borrow to purchase their houses and
government‘s national debt burden. Also, frequent changes in i/r may
not be desirable for producers in making long term investment decisions.
Thus, investment may be reduced and this will affect the productivity and
long term growth of the country.

Whether interest rates alone is effective in reducing inflation also


depends on the root cause of the inflation. If the higher expenditure is
due to higher G or lower T, then fiscal policy of reducing G and
increasing T may be a better policy in reducing inflation. On the other
hand, if the higher expenditure is due to easy availability of credit (e.g.
lower restrictions on hire purchase), then a better form of monetary
policy will be to control the availability of credit (qualitative MP) and not
interest rates.

If inflation is due to high cost of production, then interest rates failed


completely. If the cost of production is raised due to an increase in
wages faster than labour productivity, then an appropriate solution will be
to increase the labour productivity and not increase interest rates.

In a nutshell, the impact of interest rates on the general price level


Conclusion
depends on the nature of the economy and the causes of inflation.
Hence it is difficult to pinpoint whether interest rates alone is sufficient to
control inflation. Given the difficulty in identifying the root cause of
inflation, countries generally have to use a mixture of measures to bring
inflation down before it escalate out of control.

13
Level Descriptors Marks
L3 Able to explain and analyse the various policies 9-11
targeted at the different causes of inflation .
L2 Able to explain how interest rates control inflation . 6-8
however evaluations may be unclear.
L1 Poor and inaccurate explanation of how interest rate 1-5
controls inflation .
Evaluation
E2 A well developed judgment 3-4
E1 Making an undeveloped judgment 1-2

14
6 Explain the effects of globalization on the Singapore economy and assess
whether the Singapore government currently adopts appropriate policies to
minimize the costs of globalization. [25]

(1) Analysis of Question

Explain the effects of globalization on the Singapore economy and assess


whether the Singapore government currently adopts appropriate policies to
minimize the costs of globalization. [25]

Command word: Key word:


Expain requires you to interpret the meaning of globalization– Need to
globalization and its effect (positive and negative) understand the meaning of
on the Singapore globalization – it
Assess requires you to encompasses more than just
- Match and assess macroeconomic policies to greater trade to include free
costs of globalization capital flows an labour
- Consider: Are the policies effective? How
mobility and discuss with
effective or important? (Thesis + Anti-thesis)
- Consider the ―It depends‖ clause; Synthesis reference to Singapore
- Consider that your answer will depend on a context
range of factors given the context
- Evaluate policy decisions

(2) Schematic Plan (to be done in 3-4 mins)

Effects of globalization

Gains Costs

- More trade - Competition from new


- Access to overseas competitors
markets - Structural
- Higher FDIs unemployment
- FDIs competed away

Synthesis: Does gains outweigh costs?

15
Policies to minimize costs of globalization

Costs of globalization

Demand-pull FDI Diversion Structural Over-reliance


inflation unemployment on exports

Demand-management policies Supply-side policies Trade policy

Monetary policy Fiscal policy i.e. Retraining Encourage


i.e. Strong ER reduction in SMEs to go
corporate tax overseas

Evaluation: Are the policies effective?

(3) Write out the full essay

Thinking process Content / Analysis / Evaluation


Introduction Globalization is defined as ―a trend increase in mobility of products,
Define globalization services and factors of production across national borders‖ and thus
Explain the economies of the world become increasingly integrated. Such
importance of increased mobility allows economies to enhance trade [based on the
globalization to Spore theory of CA] and to access greater volume of financial, knowledge
(give some [on production methods], managerial and other resources and the
background about best practices available elsewhere — the essential technical know-
Spore). how on how to compete in the world economy. Thus, globalization
offers extensive opportunities for economic growth for open
economies, especially Singapore which is one of the most globalized
countries in the world.

Unfortunately, for the same reason that Singapore is a much more


open economy than other economies (US, China, Japan), Singapore
is more exposed to the pressures of globalization such as rising
income inequality and threatening employment and living standards.
As such, Singapore must learn to tap the gains from globalization,
while remaining realistic about its potential and its risks.

16
Thinking process Content / Analysis / Evaluation
Body - Thesis An aspect of globalization is free trade. Conventional economic
Explain the Gains theory states that a country which participates in free trade will
from globalization specialize according to the principle of comparative advantage and
export her CA good for non-CA goods. Moreover, the country can
1. Free trade [CA exploit economies of scale from larger markets and thus lower their
theory] average costs of production, making her exports more competitive
and thus boosting her exports. Such integration with the world
economy allows a country to enjoy higher economic growth and to
consume beyond her production possibility curve. This raises the
standard of living of the country‘s residents.
Analysis - Relate to This explains why globalization will work to Singapore‘s advantage as
Spore she has largely focused on international trade for economic growth.
Given her well-developed infrastructure and well-educated workforce
due to constant investment in technology and human capital
development, Singapore specializes mainly in producing and
exporting high-end intermediate manufacturing goods (electronic
goods such as data processing machines and telecommunications
equipments) in which her CA lies. Given her small domestic demand,
globalization will boost her AD via strong exports growth. This in turn
raises her national income to rise several times higher due to the
multiplier effect and allows Singapore to enjoy robust economic
Analyse the benefits growth and development, greater employment and thus lower cyclical
to her macroec. aims unemployment due to greater utilization of existing resources.

2. Access to It is also easier for Singapore to access to bigger new overseas


overseas markets markets (such as China, India, and Middle East) in the wake of
globalization. This will further boost her exports and in turn her
exports revenues, improving her balance of payments surplus. More
sources of foreign demand for her exports results in greater economic
growth and employment for the Singapore economy. Moreover, such
additional foreign demands from China and Middle East will cushion
any fall in demand for her exports by US (her main exporter) and
Evaluate the extent of make her economy more resilient in the wake of a US recession.
benefit for Singapore Given Singapore‘s ―uniquely‖ small domestic market, such access to
relative to other overseas markets allows Singapore to enjoy large economic benefits.
countries.

3. Increased Globalization also increases the availability of supplies of important


availability of raw materials and this greatly benefits Singapore which lacks natural
supplies of raw resources and is very dependent on imports. This is likely to lower her
materials and cost of production resulting in greater potential growth and lower cost-
consumer goods push inflation.

Moreover, consumers in Singapore would gain substantially in


consumer surplus due to the ability to consume increased variety of
goods and at competitive prices. It is due to consumers being able to
substitute lower-cost imports from FTA partner for higher-price
domestic goods. Such trade creation is welfare-enhancing and raises
standard of living of both countries.

17
Thinking process Content / Analysis / Evaluation
4. Higher FDIs In addition, globalization can allow Singapore to benefit as main
receiver of FDI in the region, given her sound economic policies and
political stability, compared to other Asean economies. Many firms
and MNCs in US and UK offshore their productions to Singapore to
take advantage of the lower cost of production and efficient
infrastructure. The influx of FDI (foreign capital and technologies),
coupled with the government‘s ―open-door policy‖ in attracting foreign
talent and their ideas, skills and expertise, will increase both AD and
the economy‘s productivity (thus aggregate supply) in the short and
long runs respectively. This not only accelerates but also sustains
non-inflationary economic growth (due to lower cost-push inflation) of
the country and creates more employment opportunities for the
people. Moreover, such FDI and talent enhance the labour
productivity, allowing the average worker to earn higher income,
raising his material standard of living in Singapore.

Body - Anti-thesis Although Singapore will benefit greatly from globalization due to her
Explain Costs of active participation, it is becoming more difficult to attract FDIs and
globalization foreign talents in the light of emerging competitors such as China and
India. An important aspect of globalization is the trend of firms
1. Competition from relocating their productions to other economies which offer cheaper
new competitors pools of labor and lower-cost facilities. While such a trend has
benefited Singapore in the past, it threatens her economy now. As
China gains CA in labour-intensive manufacturing goods, cost
considerations have prompted many firms, including disk-drive
makers, to move their production to China and India. As a result of
the exits of firms and the decline of manufacturing industry,
manufacturing workers, especially the less skilled, are retrenched and
caused structural unemployment due to a mismatch of skills between
jobs and labour. Singapore is much more threatened in this aspect
Analyse the extent of than her neighbouring countries as her workers become more
the downside relative expensive (higher wages) due to her faster rate of economic
to other countries. development.

2. Structural Unable to compete on cost considerations alone, Singapore has to


unemployment due move up the value chain and offer high value-added services and
to structural products such as chemicals, pharmaceutical and biotechnology in
changes in the order to enhance her competitiveness.
Spore economy
However, low-skilled manufacturing workers, without training, do not
possess the relevant skills to be employed in the new industries and
thus become structurally unemployed. Furthermore, there is widening
Evaluate – consider of wage disparity between the skilled and the unskilled as indicated
impact on different by the rising Gini coefficient (which measures income inequality in the
sectors of society i.e. economy) in Singapore. This is because skilled workers who are
low-skilled workers vs highly demanded in new industries enjoy a faster growth in wages
skilled worers while unskilled workers in declining industries saw their wages
stagnated, if not loss of wages for the structurally unemployed.

18
Thinking process Content / Analysis / Evaluation
3. FDIs competed Also, the consequences of FDIs (reduced potential growth and
away from Spore employment opportunities) being competed away are far greater for
the Singapore economy which relies more heavily (than US and
Japan) on such transfers of new knowledge and technologies by
foreign MNCs to drive her economic growth forward.

4. Increased Free capital flows (which characterized globalization) result in greater


vulnerability mobility of capital and thus increase instability and potential financial
shocks to economies. Global financial institutions and investors are
quick to pull out their funds if they sense deteriorating business
confidence. And the Singapore economy is equally vulnerable to such
capital flights. However, fortunately for Singapore, despite that her
financial market is very open to international flows, she faces
relatively less problems from the increasing instability of global
financial markets. The is due to her low levels of foreign and
corporate debts combined with its strong position in foreign reserves
– factors that helped her to remain less vulnerable to external
financial shocks than other Asian economies, as seen in the Asian
financial crisis. This allows Singapore to develop her bond market
which is important to attract global investors and to provide a source
of non-bank loans for firms and new start-ups to fund their capital
expenditure.

Explain the presence Increased trade and capital movements due to globalization have led
of the costs of to conflicts in achieving some of its micro as well as macro goals.
globalization and thus Hence, the Singapore government has put in place some demand as
policies are needed to well as supply management policies to help Singapore cope with
deal with it these problems.

Policy to deal with In view of imminent demand-pull inflation due to the strong growth in
demand-pull inflation. exports and investment (FDI), the Singapore government has put in
[Note that MP in place the exchange rate policy of adopting a gradually appreciating
Singapore is ER Singapore dollar exchange rate to keep price of imports relatively
policy.] lower, thus reducing the possibility of imported inflation and at the
same time, preventing excessive increases in export demand through
our strong Sing dollar, thus reducing excessive increases in
aggregate demand (AD) , hence reducing demand-pull inflation.

Evaluation of ER But, while our strong Sing dollar was able to help us cope with
policy inflation problems due to globalization, it may result in conflicts in
achieving macro goals between healthy BOP and short-run actual
growth. The strong Sing dollar makes imports relatively cheaper in
domestic currency and may therefore encourage further increases in
import expenditure (assuming the price elasticity of demand for
imports exceeds 1), thus worsening the BOP. Loss of export
competitiveness in the short run through the strong Sing dollar may
result in adverse effects on growth (assuming price elasticity of
demand for exports exceeds 1).

Nevertheless, Singapore‘s exchange rate policy has always been

19
Thinking process Content / Analysis / Evaluation
successful in helping Singapore cope with imported inflation and
Singapore has always been facing BOP surpluses even with the
strong SGD, as the negative impact on growth (from the loss of
export competitiveness) is insignificant in the short run as price
elasticity of demand for exports is less than 1 in the short run, and in
the long run, due to our reliance on imports of raw materials in our
production, the lower cost of production (as a result of the relatively
cheaper imported raw materials) restores our export competitiveness,
thus increasing export revenue in the long run. This policy has been
relatively successful so far since Singapore has always been facing
BOP surpluses and inflation had been kept relatively low and stable
with the help of other complementary demand and supply
management tools.

Supply-side policy to Singapore has adopted manpower policies (retraining and education
deal with structural eg. Skills Development Fund) to equip workers with new skills so as
unemployment to increase their occupational mobility and to fit them into new
industries that were created.

Singapore government also spearheads R&D projects to develop new


areas of growth and comparative advantage (CA) in the face of the
erosion of Singapore‘s CA in an attempt to search for new markets
that require employment of more workers.

Evaluation of Such supply side policies materialise only in the long run as it takes
retraining policy time to train workers and to find new CA. And the increase in G on
retraining and R&D can be inflationary in the SR. Besides, success of
retraining efforts depends on the motivational levels of workers
involved.

Policy to deal with FDI Singapore government has in place expansionary fiscal policy
diversion through the reduction in corporate and income taxes in order to
attract FDI and foreign talent. Government also increases its
spending on the provision of world-class infrastructure (physical and
telecommunications) in order to increase business competitiveness,
attracting FDI.

Evaluation of Singapore‘s fiscal policy which is very much supply-side in nature has
decreasing taxes been relatively successful in attracting FDI and foreign talent.

- Emphasis on capital accumulation through FDI and increase


in transfer of skills and expertise through foreign talent results in both
AD and AS shifting rightwards, thus achieving low inflationary
sustained growth.

- But government may incur budget deficits due to the loss in


tax revenue and higher G expenditures.

- To attract FDI, government needs to ensure that our corporate


tax rates remain competitive against that of our neighbouring

20
Thinking process Content / Analysis / Evaluation
countries eg HK.

Trade Policy to deal In view of the threat of over-reliance on exports and increased
with over-reliance on vulnerability in face of external shocks, the Singapore government
exports takes a long term view towards this issue by encouraging SMEs to
venture overseas so as to reduce the dependence of our current
account on balance of trade. With our SMEs investing abroad, these
investment incomes will in the LR increase Singapore‘s capital
account so as to balance or offset any BOT deficits due to our
reliance on exports demand in face of external shocks. It also helps to
reduce our vulnerability to changing international conditions.

Evaluation of policy to SMEs might not have the capital to venture overseas and might need
get SMEs to venture government‘s subsidies which is a drain on govt‘s budget. It is also
abroad not a guarantee that SMEs will do well overseas. A more pressing
issue is the general Singaporean is still risk-averse. Thus, the
effectiveness of trade policy which encourages local firms to venture
overseas markets depends on how quickly domestic firms and
entrepreneurs can adapt to such risk-taking mindset.

Conclusion Given the conflicts in attainment of Singapore‘s goals in the face of


Evaluation – Make a an increased integrated world, policies adopted by Singapore have
valued judgement been relatively successful in dealing with these conflicts. Our
based on evidence mixture package of demand-management, supply-side and trade
and appropriateness policies have enabled us to target our employment levels and growth
of policies for specific rate both in the short run and long run, which can be compatible if the
costs. government is able to identify the type of new industries to develop/or
expand and increase the occupational mobility of factors. Our inflation
rate is kept under control and the BOP is not a problem at present.

Level Descriptors Marks


L3 Able to explain and analyse the effects of globalization 15-21
and bring in the effectiveness of current policies to deal
with the negative impact of it with good use of economic
concepts
L2 Able to explain effects of globalization and policies govt 8-14
undertake to deal with costs of it without much
evaluation
L1 Explain only the effects of globalization, with some 1-7
mistakes.
Evaluation
E2 A well developed judgment 3-4
E1 Making an undeveloped judgment 1-2

21

You might also like